EMT FINAL EXAM

A PATIENT WHO HAS SHALLOW, SLOW IRREGULAR GASPOING BREATHS IS SAID TO HAVE _______________ RESPIRATIONS?

AGONAL

A PATIENT WITH MULTIPLE LONG-BONE FRACTURES OF THE EXTREMITIES SHOULD BE A HIGH PRIORITY FOR TRANSPORT AND SHOULD BE TREATED?

EN ROUTE TO THE HOSPITAL

A TEENAGE MALE HAS FALLEN ONTO A RAILING WHILE SKATEBOARDING. HE COMPLAINS OF RIGHT-SIDED CHEST PAIN AND MODERATE DYSPNEA. EXPOSURE OF THE CHEST REVEALS A SECTION OF HIS RIBS THA TIS MOVING OPPOSITE OF THE REST OF THE RIBS. YOU SHOULD?

APPLY A BULKY DRESSING OVER THE SECTION

A YOUNG FEMALE WAS INJURED IN AN AUTOMOBILE WRECK, STRIKING HER HEAD ON THE WINDSHIELD. SHE IS UNCONSCIOUS AND HER BREATHING IS IRREGULAR. SHE IS BLEEDING FROM A SCALP WOUND, BUT YOUR ASSESSMENT SHOWS THAT HER CRANIUM IS INTACT. HER VITAL SIGNS ARE PULSE

CLOSED HEAD INJURY

AFTER ACTIVATING THE AUTO-INJECTOR, HOW LONG SHOULD IT BE HELD IN PLACE BEFORE REMOVING IT FROM THE PATIENTS THIGH?

10 SECONDS

AN EMT INVOLVED IN AN ESPECIALLY DIFFICULT CALL, SUCH AS ONE IN WHICH A COWORKER WAS KILLED SHOULD BE URGED TO ....

SEEK HELP FROM A TRAINED MENTAL HEALTH PROFESSIONAL

AS YOU ARE TRANSPORTING A CONSCIOUS PATIENT YOU NOTICE THAT HIS BREATHING HAS BECOME LABORED. WHICH ACTION SHOULD YOU TAKE BASED ON THIS INFORMATION?

- NOTIFY THE RECEIVING FACILITY
- ENSURE A PATENT AIRWAY AND ADEQUATE O2
- CONTINUE REASSESSMENT

BARBITURATES MAY BE REFERRED TO AS WHICH OF THE FOLLOWING?

DOWNERS

BEFORE APPLYING A NRM, THE EMT SHOULD TAKE WHAT ACTION?

INFLATE THE RESERVOIR BAG 1/3 OF THE WAY

BURNS POSE A GREATER RISK TO INFANTS AND CHILDREN FOR WHICH OF THE FOLLOWING REASONS?

PEDIATRIC PATIENTS HAVE A GREATER RISK OF SHOCK FROM THE BURN

CARBON MONOXIDE POISONING OCCURS BY WHICH OF THE FOLLOWING ROUTES?

INHALATION

CHEMICAL COLD PACKS ARE CARRIED ON THE AMBULANCE FOR TREATMENT OF WHICH OF THE FOLLOWING?

MUSCULOSKELETAL INJURIES

DURING THE NEW EMPLOYEE ORIENTATION, THE TRAINING OFFICER MEETS WITH THE NEW EMTS AND EXPLAINS TO THEM THE MONTHLY TRAINING SCHEDULE AND THE CLASSES THEY MUST ATTEND FOR EMT RECERTIFICATION. ONE OF THE NEW EMTS ASKS THE TRAINING OFFICER WHY THEY HAVE TO K

EMS IS CONSTANTLY EVOLVING IN RESPONSE TO EVIDENCE BASED RESEARCH

EPINEPHRINE DELIVERED BY AUTO INJECTOR MAY BE INDICATED FOR PATIENTS WITH WHICH OF THE FOLLOWING CONDITIONS?

SEVERE ALLERGIES TO PEANUT, SHELLFISH, PENICILLIN, OR BEE STINGS

EXPERT CLINICIANS MAY USE DIFFERENT APPROACHES OF THINKING THROUGH PROBLEMS, BUT WHICH OF THE FOLLOWING WILL THEY HAVE IN COMMON?

STRONG FOUNDATION OF KNOWLEDGE

HOW MANY CERVICAL VERTEBRAE ARE THERE?

7

IF THE BABY'S UMBILICAL CORD IS NOTED TO BE WRAPPED AROUND HIS NECK AFTER THE HEAD IS DELIVERED, WHICH OF THE FOLLOWING SHOULD BE DONE?

TRY TO SLIP THE CORD OVER THE BABY'S HEAD AND SHOULDERS

IN ASSESSING A PATIENT'S BREATHING, WHAT IS YOUR FIRST QUESTION?

ARE THEY BREATHING?

IN REASSESSMENT, WHAT ARE THE 3 INTERVENTIONS THAT YOU AS THE EMT SHOULD CHECK?

- OXYGENATION
- BLEEDING MANAGEMENT
- ADEQUACY OF OTHER INTERVENTIONS

IN THE PRIMARY ASSESSMENT, WHICH OF THE FOLLOWING IS NOT AN ACCEPTABLE METHOD OF ASSESSING THE PATIENT'S CIRCULATORY STATUS?

TAKING A BLOOD PRESSURE READING

IN WHICH OF THE FOLLOWING SITUATIONS HSOULD AN EMT WITHHOLD RESUSCITATIVE MEASURES FROM A PATIENT IN CARDIAC ARREST?

THE PATIENT'S CAREGIVER PRESENTS A DNR ORDER SIGNED BY THE PATIENT AND HIS PHYSICIAN

IN WHICH OF THE FOLLOWING ZONES IS THE EMT EXPECTED TO BE STAGES AT THE SCENE OF A HAZARDOUS MATERIALS INCIDENT?

COLD ZONE

MRS THOMAS CALLS 911 FOR HER 9 MONTH OLD BABY, ELIZA, WHO IS LISTLESS AND PALE. SHE TELLS YOU THAT THE BABY OVERSLEPT THIS MORNING AND HSE HAD A HARD TIME WAKING HER UP. MRS THOMAS IS VERY DISTRAUGHT WORRYING ABOUT THE BABY. WHAT SHOULD YOU DO NEXT?

TRY TO GET SOME CURRENT HISTORY FROM THE MOTHER

OPERATIONS OF EMERGENCY VEHICLES MUST DRIVE WITH THE SAFETY OF OTHERS IN MIND. THIS IS KNOWN AS....

DRIVING WITH DUE REGARD

REGARDING THE USE OF EMERGENCY SIRENS, WHICH OF THE FOLLOWING IS FALSE?

AMBULANCE OPERATIONS ARE NOT AFFECTED BY SIREN NOISE

HOW OFTEN SHOULD THE EMT REASSESS THE VITAL SIGNS OF AN UNSTABLE PATIENT?

EVERY 5 MINUTES

THE TRENDELENBURG POSITION WOULD BE USED TO TRANSPORT WHAT TYPE OF PATIENT?

A PATIENT IN SHOCK

WHAT IS THE ADEQUATE CIRCULATION OF BLOOD THROUGHOUT THE BODY, WHICH FILLS THE CAPILLARIES AND SUPPLIES THE CELLS AND TISSUES OXYGEN AND NUTRIENTS?

PERFUSION

FAINTING

SYNCOPE

THE MEDICATIONS THAT TRANSPLANT PATIENTS NEED TO TAKE FOR THE REST OF THEIR LIVES TO PREVENT ORGAN REJECTION ALSO OFTEN LEAD TO HIGH SUSCEPTIBILITY OF...

INFECTION

THE MUSCULAR ORGAN IN WHICH A BABY DEVELOPS DURING PREGNANCY?

UTERUS

THE OXYGEN FLOW RATE FOR A NASAL CANNULA SHOULD NOT EXCEED HOW MANY LPM?

6

THE PATHOPHYSIOLOGY OF __________________ IS ONE IN WHICH THE PERICARDIAL SAC FILLS WITH BLOOD TO THE POINT WHERE THE CHAMBERS OF THE HEART NO LONGER FILL ADEQUATELY, USUALLY SECONDARY TO TRAUMA.

CARDIAC TAMPONADE

THE TRANSITION FROM CHILDHOOD TO ADULT IS KNOWN AS WHICH OF THE FOLLOWING?

ADOLESCENT

TO BE COMPASSIONATE AND EMPATHETIC, TO BE ACCURATE WITH INTERVIEWS, AND ATO INSPIRE CONFIDENCE ARE ALL EXAMPLES OF WHICH OF THE PERSONAL TRAITS OF A QUALITY EMT?

ABLE TO LISTEN TO OTHERS

TO EXHALE, THE DIAPHRAGM AND INTERCOSTAL MUSCLES RELAX TO CONTRACT THE CHEST, WHICH CREATES A POSITIVE PRESSURE. THE IS WHAT TYPE OF PROCESS?

PASSIVE

TRADITIONALLY AN APGNAR SCORE IS TAKEN AT WHAT TIME INTERVALS ARE BIRTH?

1 AND 5 MINUTES

TWO CHRONIC MEDICAL CONDITIONS THAT DIALYSIS PATIENTS FREQUENTLY HAVE IN ADDITION TO KIDNEY FAILURE ARE _________ AND __________.

HYPERTENSION
DIABETES

WHAT BEST DEFINES THE EVALUATION OF THE PATIENT'S CONDITION IN ORDER TO PROVIDE EMERGENCY CARE?

PATIENT ASSESSMENT

WHAT ARE THE SIGNS OF HYPOXIA?

COMMONLY SEEN AS BLUE OR GRAY SKIN, DETERIORATION OF PATIENT'S MENTAL STATUS LIKE CONFUSION OR RESTLESSNESS

WHAT IS THE EMTS HIGHEST PRIORITY AT THE SCENE OF A HAZARDOUS MATERIALS INCIDENT?

PERSONAL SAFETY

WHAT IS ANOHER NAME FOR AN ALLERGIC REACTION?

ANAPHYLAXIS

WHAT IS THE CORRECT ORDER OF AIR FLOW FROM THE NOSE TO THE ALVEOLI IN THE LUNGS

NOSE
NASOPHARYNX
PHARYNX
HYPO PHARYNX
LARYNX
TRACHEA
BRONCHI
ALVEOLI

WHAT IS THE PRIMARY EFFECT ON THE BODY WHEN AN EMT ASSISTS A PATIENT WITH A PRESCRIBED INHALER IF THE PATIENT IS SHORT OF BREATH?

RELAXATION OF THE BRONCHIOLES

WHAT IS IT CALLED WHEN A PATIENT DESCRIBES HOW HE FEELS?

HIS SYMPTOMS

WHEN CARBON DIOXIDE IS NOT EXCHANGED, THE NET RESULT IS HIGH CARBON DIOXIDE, A CONDITION CALLED _____________, WITHIN THE BODY.

HYPERCAPNIA

WHEN CHECKING BREATH SOUNDS IN A TRAUMA PATIENT, WHAT SHOULD THE MET ASSESS FOR FIRST?

PRESENCE AND EQUALITY

WHEN CONTROLLING THE SCENE OF AN EMERGENCY INVOLVING DAMAGED POWER POLES AND DONED ELECTRICAL WIRES, THE EMT MUST REMEMBER TO:

REQUEST APPROPRIATE PERSONNEL TO CUTE THE POWER SOURCE

WHEN DOES RESPIRATORY DISTRESS CHANGE TO RESPIRATORY FAILURE?

WHEN THE RESPIRATORY CHALLENGE CONTINUES, THE SYSTEMS CANNOT KEEP UP WITH THE DEMAND, AND SKIN COLOR AND MENTAL STATUS CHANGE

WHEN GIVING A RADIO REPORT TO THE HOSPITAL, WHICH OF THE FOLLOWING SHOULD BE STATED FIRST?

THE PATIENT IS A 55 Y.O. MALE

WHEN GROSSLY DECONTAMINATING A PATIENT WHO IS NOT WEARING PPE IN A HAZARDOUS MATERIALS INCIDENT, WHICH OF THE FOLLOWING TECHNIQUES SHOULD BE USED?

TAKING A LOW PRESSURE DECONTAMINATION SHOWER

WHEN PREPARING TO MOVE A PATIENT THAT IS USING A SPECIALTY MEDICAL DEVICE FROM HIS HOUSE TO THE AMBULANCE, WHICH OF THE FOLLOWING QUESTIONS WOULD BE APPROPRIATE TO ASK THE FAMILY?

- WHAT WORKED BEST FOR MOVING THE PATIENT THE LAST TIME HE WAS TRANSPORTED BY AMBULANCE?
- HOW DO YOU NORMALLY MOVE THE PATIENT?

WHAT DO DIALATED PUPILS LOOK LIKE?

LARGER THAN NORMAL

WHAT IS IT CALLED WHEN THE HEART CONTRACTS AND FORCES BLOOD INTO THE ARTERIES, THE PRESSURE IS CREATED?

SYSTOLIC BLOOD PRESSURE

WHEN USING THE AED, WHICH OF THE FOLLOWING PEOPLE IS RESPONSIBLE FOR CALLING TO "CLEAR" THE PATIENT BEFORE DELIVERING A SHOCK?

THE EMT OPERATING THE DEFIBRILLATOR

WHEN YOU PLACE YOUR FINGER IN AN INFANT'S PALM, SHE GRASPS IT WITH HER FINGERS. THIS IS AN EXAMPLE OF WHICH REFLEX?

PALMAR REFLEX

WHERE DO BASELINE VITAL SIGNS FIT INTO THE SEQUENCE OF PATIENT ASSESSMENT?

AT SECONDARY ASSESSMENT

WHICH ABDOMINAL QUADRANT CONTAINS THE APPENDIX?

RIGHT LOWER

WHICH OF THE FOLLOWING BEST DESCRIBES STATUS EPILEPTICUS?

TWO OR MORE SEIZURES WITH TONIC CLONIC ACTIVITY WITHOUT AN INTERVENING PERIOD OF CONSCIOUSNESS

WHICH OF THE FOLLOWING BEST DESCRIBES THE EMTS INTENDED ROLE IN THE IDEAL PROVISIONS OF EMERGENCY CARDIAC CARE?

EARLY DEFIBRILLATION

WHICH OF THE FOLLOWING BEST DESCRIBES THE DELIVERY OF O2 AND NUTRIENTS AT THE BODY'S CELLULAR LEVEL?

PERFUSION

WHICH OF THE FOLLOWING BEST DESCRIBES THE MEDICAL CONDITION OF SHOCK?

A STATE OF INADEQUATE TISSUE PERFUSION

WHICH OF THE FOLLOWING BEST DESCRIBES THE PURPOSE OF THE ODERN EMERGENCY MEDICAL SERVICES (EMS) SYSTEM?

TO HAVE TRAINED PERSONNEL RESPOND QUICKLY AND PROVIDE EMERGENCY CARE ON THE SCENE, DURING TRANSPORT, AND AT THE HOSPITAL

WHICH OF THE FOLLOWING CAUSES WORSENING OF THE DAMAGE IN A BRAIN INJURY?

IMPROPER MANAGEMENT OF AIRWAY AND VENTILATION

WHICH OF THE FOLLOWING CHEMICAL AGENTS PROHIBITS THE USE OF OXYGEN IN THE CELLS?

CYANIDE

WHICH OF THE FOLLOWING DESCRIBES AN OPEN EXTREMITY INJURY?

BONE TISSUES HAVE LACERATED FROM THE INSIDE

WHICH OF THE FOLLOWING DESCRIBES THE MIDLINE OF THE BODY?

AN IMAGINARY LINE DIVIDING THE BODY INTO EQUAL RIGHT AND LEFT HALVES

WHICH OF THE FOLLOWING DESCRIBES THE NORMAL APPEARANCE OF AMNIOTIC FLUID?

CLEAR AND COLORLESS FLUID

WHAT ARE THE CONSIDERATIONS FOR LIFTING?

...

WHICH OF THE FOLLOWING INHALERS WOULD NOT BE USED TO REVERSE AN ASTHMA ATTACK?

BECLOMETHOSONE

WHICH OF THE FOLLOWING IS NOT A CLASSIFICATION OF THE SEVERITY OF SHOCK?

UNCOMPENSATED

WHICH OF THE FOLLOWING IS FALSE REGARDING THE PURPOSE OF IMMEDIATELY DOCUMENTING VITAL SIGNS ONCE THEY ARE OBTAINED?

FAILURE TO RECORD IS FALSIFYING THE RECORD

WHICH OF THE FOLLOWING IS NOT A BENEFT OF SPLINTING AN INJURY TO BONES AND CONNECTIVE TISSUES?

RESTRICT BLOOD FLOW TO THE SITE OF INJURY TO PREVENT SWELLING

WHICH OF THE FOLLOWING IS NOT A REASON THAT AN AED MAY INDICATE THAT THERE IS "NO SHOCK ADVISED"?

VENTRICULAR FIBRILLATION

YOU ARE TREATING A 54 Y.O. FEMALE PATIENT WHO WAS INVOLVED IN A DOMESTIC DISPUTE; YOU NOTICE AN ABRASION TO THE SIDE OF HER HEAD. THE PATIENT IS UNRESPONSIVE WITH A BLOOD PRESSURE OF 200/110, A PULSE OF 60 BEATS PER MINUTE, SLIGHTLY IRREGULAR BREATHING. T

INTERCRANIAL PRESSURE

YOU ARE TREATING A 57 Y.O. MALE FOR CHEST PAIN. YOU HAVE GATHERED ALL PERTINENT HISTORY OF PRESENT ILLNESS, COMPLETED TWO SETS OF VITALS, TALKED WITH MEDICAL DIRECTION, AND ASSISTED THE PATIENT WITH TWO DOESES OF HIS NITROGLYCERIN. DETERMINATION OF WHETHE

REASSESSMENT

YOU ARE TREATING A PATIENT WHO HAS A PRODUCTIVE COUGH AND WHO REPORTS WEIGHT LOSS, LOSS OF APPETITE, WEAKNESS, NIGHT SWEATS, AND FEVER. WHICH OF THE FOLLOWING DISEASES IS MOST LIKELY THE CAUSE OF THE SE SIGNS AND SYMPTOMS?

TUBERCULOSIS

YOU ARRIVE ON THE SCENE OF THE 55 Y.O. MALE PATIENT. THE PATIENTS WIFE CALLED 911 BECAUSE HE IS HAVING CHEST PAINS. THE PATIENT IS VERY ANGRY WITH HIS WIFE FOR CALLING 911 BECAUSE HE STATES HE ONLY HAS HEARTBURN AND ADAMANTLY REFUSES ANY TREATMENT OR TRAN

PROVIDE EMERGENCY CARE UNDER IMPLIED CONSENTS

YOU FIND A TEENAGE MALE LYING SUPINE IN HIS BEDROOM. YOU HEAR GURGLING SOUNDS FROM THE PATIENT'S MOUTH AND SEE VOMIT WITH PILL FRAGMENTS ON THE FLOOR. YOU SHOULD:

SUCTION THE AIRWAY

YOU HAVE ARRIVED AT THE SCENE OF A CALL A "MAN DOWN." AS YOU ENTER THE RESIDENCE YOU NOTE THAT YOUR PATIENT IS MALE IN HIS MID-60S WHO IS AWAKE BUT DOESN NOT SEEMS TO ACKNOWLEDGE YOU PRESENCE. HE IS PERSPIRING PROFUSELY, HAS CYANOSIS OF HIS EARS AND LIPS,

CHECK FOR A RADIAL PULSE

YOU HAVE JUST ADMINISTER NITROGLYCERIN TO A 68 Y.O. PATIENT. WITHIN A FEW MINUTES, SHE COMPLAIN OF FEELING FAINT AND LIGHTHEADED, BUT STATES THAT SHE IS STILL HAVING SOME CHEST PAIN. WHICH OF THE FOLLOWING WOULD BE THE BEST SEQUENCE OF ACTIONS?

LOWER THE HEAD OF THE STRETCHER AND TAKE THE PATIENTS BLOOD PRESSURE

A 5 year old boy complains of pain to the right lower quadrant of his abdomen. Correct assessment of this child's abdomen includes

palpating the left upper quadrant first

A 6 year old male, who weighs 40 pounds, ingested a bottle of aspirin approximately 20 minutes ago. Medical control orders you to administer activated charcoal in a dose of 1 g/kg. How much activated charcoal will you administer?

18 g

A 9-year-old girl was struck by a car while she was crossing the street and is displaying signs of shock. During your assessment, you note a large contusion over the left upper quadrant of her abdomen. Which of the following organs has most likely been in

Spleen

A 30 year old woman has sever lower abdominal pain and light vaginal bleeding. She tells you that her last menstrual period was 2 months ago. On the basis of these findings, you should suspect

an ectopic pregnancy

A 40-year old patient sustained full=thickness burns to the entire head, anterior chest, and both anterior upper extremities. Using the adult Rule of Nines, what percentage of the patient's body surface area has been burned?

27%

A 42- year old man was ejected from his car after it struck a bridge pillar at a high rate of speed. You find him lying approximately 50' from the car. After manually stabilizing his head, your next action should be to

grasp the angles of the jaw and lift

A 56-year old man has labored, shallow breathing at a rate of 28 breaths/min. He is conscious, but extremely restless. Airway management should consist of

positive pressure ventilation

A 56-year-old man with a history of cardiac problems reports pain in the upper midabdominal area. This region of the abdomen is called the

epigastrium

A 60-year-old woman is experiencing severe respiratory distress. When you ask her a question, she can only say two words at a time. You should manage this patient by

providing positive pressure ventilations.

A 75 year old man has generalized weakness and chest pain. He has a bottle of prescribed nitroglycerin and he states that he has not taken any of his medication. After initiating oxygen therapy, you should next

complete a focused physical examination, including obtaining baseline vital signs.

An 80-year old woman has pain in the right upper quadrant of her abdomen and a yellowish tinge to her skin. You should suspect dysfunction of the

liver

The AED analyzes your pulseless and apneic patient's cardiac rhythm and advises that a shock is indicated. You should

ensure that nobody is touching the patient

After an initial attempt to ventilate a patient fails, you reposition the patient's head and reattempt ventilation without success. You should next

initiate airway obstruction removal techniques and provide transport

After assisting a patient with her epinephrine auto-injector, you should dispose of the device by

placing the device in a puncture proof container

After removing a patient from the water, your assessment reveals that the patient is breathing inadequately and is continuously regurgitating large quantities of water. You should manage this patient by

alternating suctioining with artificial ventilations

After the delivery of the defibrillation with the AED, the patient has a return of a pulse. You should next

assess the airway and ventilatory status

As you are performing CPR on an elderly man, his wife presents you with a "do not resuscitate" order. Your most appropriate course of action is to

continue CPR until medical control is notified

As you are providing initial ventilations to a patient with apnea using a bag-valve-mask device, you note minimal rise of the chest. You should next

increase the volume of the ventilations

As you assess a 56-year old man, you note that he is pulseless and apneic. As your partner gets the AED from the ambulance, you should

perform CPR until the AED is ready to use

As you begin your assessment of an unresponsive man who fell approximately 20' from a roof, you should first

manually stabilize the patient's head and perform a jaw-thrust maneuver

As you step out of the ambulance at the scene of a nighttime motor vehicle crash on the highway, your immediate concern should be

oncoming traffic

At the peak of the inspiratory phase, the alveoli in the lungs contain

more oxygen than carbon dioxide

At the scene of a mass-casualty incident, you notice a bystander who is clearly emotionally upset. An appropriate action to take would be to

assign the bystander a simple, non-patient-care task

The automated external defibrillator (AED) should NOT be used in patients who

are apneic and have a weak carotid pulse

An awake and alert 92-year-old woman with chest pain is refusing EMS treatment and transport to the hospital. Her family insists that you transport her. This situation is most appropriately managed by

advising the patient of the risks of refusing care

A common side effect of nitroglycerin is

headache

During a bar fight, a 22-year old man was stabbed in the chest with a large knife. The patient is pulseless and apneic, and the knife is impaled in the center of his chest. management should include

removing the knife, starting CPR, and providing rapid transport

During a soccer game, an 18 year old woman injured her knee. You note that the knee is in the flexed position and is obviously deformed. Your first action should be to

manually stabilize the leg above and below the knee

During the initial assessment of a trauma patient, you note massive facial injuries, weak radial pulses, and clammy skin. What should be your most immediate concern?

Potential obstruction of the airway

During the rapid trauma assessment of a patient with multiple injuries, you expose the chest and find an open wound with blood bubbling from it. What should you do next?

Prevent air from entering the wound.

During your assessment of a 34-year old man with a gunshot wound to the chest, you note that his skin is pale. This finding is most likely caused by

decreased blood flow to the skin

During your initial assessment of an unconscious adult patient, you find the patient is apneic. You should next

deliver two rescue breaths

The effectiveness of chest compressions are most effectively assessed by

palpating for a carotid pulse with each compression

An elderly man is found unconscious in his kitchen. The patient's wife tells you that her husband has diabetes and that he took his insulin, but did not eat anything. You should suspect

hypoglycemia

An EMT-B's failure to obtain consent to treat a patient could result in allegations of

battery

Firefighters have rescued a man from his burning house. he is conscious and in considerable respiratory distress. He has a brassy cough and singed nasal hairs. The most immediate threat to this patient's life is

closure of the airway

Following an apparent febrile seizure, a 4 year old boy is alert and crying. His skin is warm and moist. The most appropriate management of this child includes

offering oxygen and providing transport

Following blunt injury to the anterior trunk, a patient is coughing up bright red blood. You should be most suspicious of

bleeding within the lungs

Following delivery of a newborn, you note that the mother has a moderate amount of vaginal bleeding. The mother is conscious and alert and her vital signs are stable. The most appropriate management of the mother includes

administering oxygen and massaging the uterus

Immediately following a generalized motor seizure, most patients are

confused

Immediately upon delivery of a newborn's head, you should first

suction the mouth

In a patient with cardiac compromise, you would be LEAST likely to encounter

headache

In addition to ensuring your own safety, your primary responsibility when functioning at the scene of a violent crime is to

appropriately manage the patient

In most states; the EMT-Basic is required to report which of the following occurrences?

Animal bite

In the patient with diabetes, hypoglycemia typically presents with

clammy skin and a rapid onset

In which of the following patients would an oropharyngeal airway be indicated?

An unconscious patient with fluid drainage from the ears

In which of the following patients would nitroglycerin be contraindicated?

53-year old male with chest discomfort, diaphoresis, a blood pressure of 146/66 mm Hg, and regular use of Cialis

Indications that artificial ventilations in an apneic adult are ineffective include

asymmetrical rise of the chest

Initial attempts at providing artificial ventilation should be accomplished using

a pocket mask with supplemental oxygen

the initial treatment of choice for ventricular fibrillation of short duration, such as a witnessed cardiac arrest is

prompt defibrillation

Management of a patient with severe abdominal pain includes

administering 100% oxygen

Management of an 18-year old woman with severe vaginal bleeding includes all of the following, EXCEPT

placing sterile dressings into the vagina

Management of an unconscious, breathing patient with a significant cardiac history would include all of the following, EXCEPT

analyzing the rhythm with an AED

A middle-aged woman has acute shortness of breath and respirations of 30 breath/min. How should you first manage this patient?

Assess respiratory quality

The middle, muscular layer of the heart is called the

myocardium

The most appropriate management of a patient who has sustained widespread full-thickness burns following an explosion should consist of applying

oxygen; dry, sterile dressings; warmth; and providing rapid transport

The most effective means of preventing the spread of disease is

effective handwashing

The most effective method for determining whether you are providing adequate artificial ventilation is

assessing the chest for adequate rise

the most important initial steps of assessing and managing a newborn include

clearing the airway and keeping the infant warm

Patients with closed head injuries often have pupillary changes and

hypertension

The position of comfort for a patient with nontraumatic chest pain most commonly is

semisitting

Prescribed inhalers, such as albuterol (Ventolin), relieve respiratory distress by

relaxing the smooth muscle of the bronchioles.

Prevention of cardiac arrest in infants and small children should focus primarily on

providing airway management

Prior to administering nitroglycerin to a patient with chest pain, you must

contact medical control and obtain proper authorization

Prior to applying a nonrebreathing mask on a patient with difficult breathing you should

prefill the reservoir bag to ensure delivery of 100% oxygen

Prior to your arrival at the scene, a near-drowning victim was removed from the water. You should manage the patient's airway appropriately while considering the possibility of

spinal injury

Pulmonary surfactant serves which of the following functions?

It facilitates the transport of oxygen-poor blood from the right ventricle to the lungs.

The scene size up includes all of the following components, EXCEPT

applying personal protective gear

A set of regulations and ethical considerations that define the extent or limits of an EMT-Basic's job is called

scope of practice

Signs of inadequate breathing in an unconscious patient include

a rapid respiratory rate

Snoring respirations are most rapidly managed by

correctly positioning the head

A soft-tissue injury that results in a flap of torn skin is referred to as

an avulsion

A sudden onset of respiratory distress in a 5 year old child with no fever most likely is the result of

obstruction of the airway by a foreign body

To ensure delivery of the highest concentration of oxygen to your patient using a nonrebreathing mask, you should

make sure that the reservoir bag is pre inflated

To obtain the most reliable assessment of a patient's tidal volume, you should

look at the rise of the chest

Unconsciousness, shallow breathing, and constricted pupils are most indicative of what type of drug overdose?

Narcotic

Upon arriving at the scene of a multiple vehicle crash, you can see that at least two patients have been ejected from their vehicles. What should you do next?

Call for at least one more ambulance

Upon delivery of a baby's head, you see that the umbilical cord is wrapped around its neck. You should manage this situation initially by

trying to remove the cord from around the neck.

The wall that separates the left and right sides of the heart is the

septum

When assessing a patient with a complaint of chest pain, which of the following questions would you ask to assess the R in OPQRST?

Is the pain in one place or does it move around?

When assessing a patient with a reduction in tidal volume, you would expect the respirations to be

shallow

When dealing with an emotionally disturbed patient, you should be concerned with

whether the patient could harm you

When is it most appropriate to clamp and cut the umbilical cord?

As soon as the cord stops pulsating

When is the best time to perform a detailed physical examination?

While enroute to the hospital

When monitoring a patient with a head injury, the most reliable indicator of his or her condition is the

level of consciousness

Which artery should you palpate when assessing for a pulse in an unresponsive 6-month old patient?

Brachial

Which of the following actions is most important when immobilizing a patient with a suspected spinal injury?

Select and apply the appropriate size of extrication collar.

Which of the following actions should be carried out during the initial assessment of an unconscious patient?

Assessing the skin

Which of the following actions would most likely cause a sudden drop in a patient's blood glucose level?

Taking too much prescribed insulin

Which of the following assessment findings would LEAST suggest cardiac compromise?

Palpable pain to the chest

Which of the following assessment findings would most suggest a systemic reaction following ingestion of a poison?

Tachycardia and hypotension

Which of the following bones is affected with a swollen, painful deformity to the lateral bone of the left forearm?

Radius

Which of the following conditions would most likely cause flushed skin?

Exposure to heat

Which of the following describe the MOST appropriate method of performing chest compressions on an adult patient in cardiac arrest?

Compress the chest to a depth of 1 1/2" to 2", allow full recoil of the chest after each compression, minimize interruptions in chest compressions

Which of the following describes the most correct method for inserting a nasopharyngeal airway

Insert the device with the bevel facing the septum

Which of the following injuries or conditions should be managed first?

Bleeding within the oral cavity

Which of the following mechanisms cause respiratory and circulatory collapse during anaphylactic shock?

Bronchoconstriction and vasodilation

Which of the following natures of illness is most consistent with a patient with low blood glucose level who is acting bizarre and breathing shallowly?

Altered mental status

Which of the following organs is not part of the endocrine system?

Gallbladder

Which of the following parameters would be most reliable as an indicator of perfusion in a 1 year old child?

capillary refill

Which of the following patients with diabetes should receive oral glucose?

A confused patient who has cool, clammy skin

Which of the following patients would be at most risk for suicide?

A man who is in the midst of losing a significant relationship

Which of the following patients would be most in need of a rapid trauma assessment?

an awake and alert 19-year-old man with a small caliber gunshot wound to the abdomen

Which of the following patients would MOST likely present with atypical signs and symptoms of acute myocardial infarction?

72-year old female with diabetes and hypertension

Which of the following signs would LEAST suggest a diabetic emergency?

Bradycardia

Which of the following situations is an example of abandonment?

A first responder assumes patient care from an EMT-Intermediate

Which of the following situations would necessitate treatment using implied consent?

A 65-eyar old man who is semiconscious and suspected of having a severe stroke

Which of the following statements best describes a mass-casualaty incident?

The number of patients overwhelms your resources

Which of the following statements regarding the function of insulin is most correct?

It facilitates the uptake of glucose from the bloodstream into the cell.

Which of the following structures is the primary pacemaker, which sets the normal rate for the heart?

Sinoatrial node

Which position is most appropriate for a mother in labor with a prolapsed umbilical cord?

Supine with hips elevated

Which vital sign is the best indicator of cardiac output during the initial assessment?

Pulse rate and quality

While a man was using a chainsaw to trim branches from a tree, it slipped and caused a large laceration to his left forearm. Bright red blood is spurting from the wound. The patient is conscious, alert, and talking. You should first

control the active bleeding

While assessing a patient with chest pain, you note that the patient's pulse is irregular. This most likely indicates

abnormalities in the heart's electrical conduction system

While managing a patient in cardiac arrest, you turn the AED on and attach the pads to the patient. When you push the analyze button, the machine signals "low battery" and then ceases to function. The patient subsequently dies. Which of the following stat

You and your partner may be held liable for negligence.

While managing a patient with acute shortness of breath, you prepare and apply a nonrebreathing mask set at 12 L/min. The patient pulls the mask away from his face, stating that it is smothering him. You should next

reassure the patient and apply a nasal cannula instead

You are administering oxygen to a woman with asthma who took two puffs of her prescribed inhaler without relief prior to your arrival. Your next action should be to

contact medical control for further advice

You are at the scene where a man panicked while swimming in a small lake. As you attempt to rescue this patient, you should first

throw a rope to the patient

You are called to a local park for a 7-year old boy with respiratory distress. During your assessment, you find that the patient is wheezing and has wide-spread hives and facial edema. What should you suspect has occurred?

allergic reaction

You are called to treat a male patient who overdosed on heroin and and is unconscious with shallow breathing and cyanosis to the face. The patient suddenly begins to vomit. What should you do first?

Turn the patient onto his side.

You are dispatched to the scene of a motorcycle crash in which two patients were injured. Upon arrival, you find that one patient, a 19-year-old woman, is conscious and alert and is being tended to by a police officer for minor scrapes and cuts. The secon

stabilize his head manually

You are dispatched to the scene of a motorcycle crash in which two patients were injured. Upon arrival, you find that one patient, a 19-year-old woman, is conscious and alert and is being tended to by a police officer for minor scrapes and cuts. The secon

advise the patient that she should be transported to the hospital because of the seriousness of the crash

You are dispatched to the scene of a motorcycle crash in which two patients were injured. Upon arrival, you find that one patient, a 19-year-old woman, is conscious and alert and is being tended to by a police officer for minor scrapes and cuts. The secon

palpable pain at the site of the injury.

You are dispatched to the scene of a motorcycle crash in which two patients were injured. Upon arrival, you find that one patient, a 19-year-old woman, is conscious and alert and is being tended to by a police officer for minor scrapes and cuts. The secon

repeat the initial assessment and treat as needed

You are dispatched to the scene of a motorcycle crash in which two patients were injured. Upon arrival, you find that one patient, a 19-year-old woman, is conscious and alert and is being tended to by a police officer for minor scrapes and cuts. The secon

Perform a rapid trauma assessment

You are managing a conscious patient who you believe is having an acute ischemic stroke. After administering oxygen, your next priority should include:

providing prompt transport for possible fibrinolytic therapy.

You are responding to a call for a 2 year old child who fell from a second story window. With the mechanism of injury and the age of the patient in mind, you should suspect that the primary injury occurred to the child's

head

You arrive at a residence where you find a man lying unconscious in his front yard. There were no witnesses to the event that caused the unconsciousness. In assessing this man, you must assume that he

has sustained an injury

You arrive at the scene shortly after a 3-year old female experienced a seizure. The child, who is being held by her mother, is conscious and crying. The mother tells you that her daughter has been ill recently and has a temperature of 102.5 degrees F. Wh

Oxygen via the blow-by technique, remove clothing and cool the child with towels soaked in tepid water, and transport.

You arrive at the scene shortly after a 55-year-old man collapsed. Two bystanders are performing CPR. The man's wife states that he had cardiac by-pass surgery approximately 6 months earlier. There are no signs of trauma. After you attach the AED and anal

Ventricular fibrillation

You arrive at the scene shortly after a 55-year-old man collapsed. Two bystanders are performing CPR. The man's wife states that he had cardiac by-pass surgery approximately 6 months earlier. There are no signs of trauma. Cardiac arrest in the adult popul

cardiac arrhythmias

You arrive at the scene shortly after a 55-year-old man collapsed. Two bystanders are performing CPR. The man's wife states that he had cardiac by-pass surgery approximately 6 months earlier. There are no signs of trauma. Your first action in the manageme

stop CPR so you can assess pulse and breathing

You arrive at the scene where a 49 year old woman is found semiconscious on the floor of her living room. The patient's husband tells you that they were watching TV when this condition suddenly developed. No trauma was involved. The patient moans occasion

decreased tidal volume

You arrive at the scene where a 49 year old woman is found semiconscious on the floor of her living room. The patient's husband tells you that they were watching TV when this condition suddenly developed. No trauma was involved. The patient moans occasion

a decrease in the amount of arterial oxygen

You arrive at the scene where a 49 year old woman is found semiconscious on the floor of her living room. The patient's husband tells you that they were watching TV when this condition suddenly developed. No trauma was involved. The patient moans occasion

assisted ventilations with 100% oxygen

You arrive at the scene where a 49 year old woman is found semiconscious on the floor of her living room. The patient's husband tells you that they were watching TV when this condition suddenly developed. No trauma was involved. The patient moans occasion

insert a nasopharyngeal airway

You assess a newborn with cyanosis to the chest and face and a heart rate of 90 beats/min. What should you do next?

Begin artificial ventilations

You have applied a pressure bandage and additional dressings to a large laceration with severe arterial bleeding. The bandages are quickly blood-soaked. You should next

remove the bandages and apply pressure at the site of the bleeding

You have completed your prehospital care report and left a copy at the hospital when you realize that you forgot to document a pertinent finding on the front of the report. Your most appropriate action would be to

attach an addendum to the original run report

you receive a call for a 3 year old girl with respiratory distress. When you enter her residence, you see the mother holding the little girl, who does not acknowledge your presence. This finding indicates that the child

has hypoxia

You receive a call to a local daycare center for a 3-year old boy who is not breathing. When you arrive and assess the child, you find him to be in cardiopulmonary arrest. You initiate CPR and request a back-up ambulance. A paramedic unit arrives at the s

8 to 10 breaths/min

You receive a call to a local daycare center for a 3-year old boy who is not breathing. When you arrive and assess the child, you find him to be in cardiopulmonary arrest. You initiate CPR and request a back-up ambulance. After attaching the AED, you push

immediately perform CPR

You receive a call to a local daycare center for a 3-year old boy who is not breathing. When you arrive and assess the child, you find him to be in cardiopulmonary arrest. You initiate CPR and request a back-up ambulance. As you are performing one-rescuer

30:2

You receive a call to a local daycare center for a 3-year old boy who is not breathing. When you arrive and assess the child, you find him to be in cardiopulmonary arrest. You initiate CPR and request a back-up ambulance. In infants and children, the most

decreased ventilatory volume

You receive a call to a restaurant where a 34 year old man is experiencing shortness of breath. When you arrive you immediately note that the man has urticaria on his face and arms. He is in obvious respiratory distress, but is awake and alert.
Epinephrin

As a bronchodilator, it facilitates adequate breathing

You receive a call to a restaurant where a 34 year old man is experiencing shortness of breath. When you arrive you immediately note that the man has urticaria on his face and arms. He is in obvious respiratory distress, but is awake and alert.
Suspecting

place a nonrebreathing mask set at 15 L/min on the patient.

You receive a call to a restaurant where a 34 year old man is experiencing shortness of breath. When you arrive you immediately note that the man has urticaria on his face and arms. He is in obvious respiratory distress, but is awake and alert.
the patien

provide transport and consider an ALS rendezvous

You respond to a call for a shooting at a local bar. You arrive at the scene and find a young man sitting against the wall, screaming in pain, with bright red blood spurting from a wound near his groin. What should you do first?

Apply pressure to the wound

you should suspect potential abuse of a 4 year old child when you encounter

purple and yellow bruises to the thighs

Your first action in managing a patient with an altered mental status should be to

make sure that the patient is breathing adequately

_______OVERDOSE IS NOT ASSOCIATED WITH HYPERVENTILATION

NARCOTIC OD

_____MY BE THE LAST MEASURABLE FACTOR TO CHANGE IN SHOCK

BLOOD PRESSURE

THE ____OF A BULLET HAS THE GREATEST IMPACT ON PRODUCING INJURY

SPEED

THE ACTUAL EXCHANGE OF 02 AND C02 OCCURS IN THE

ALVEOLAR SACS

THE PROCESS OF BINDING OR STICKING TO A SURGACE (ACTIVATED CHARCOAL)

ADSORPTION

AN AED DELIVERS PROMPT DEFIBRILLATION TO PATIENTS WITH

VENTRICULAR FIBRILLATION

AFTER ASSISTING WITH NITRO...

REASSESS BP WITHIN 5 MINS TO DETECT HYPOTENSION

AMI OCCURS WHEN

MYOCARDIAL TISSUE DIES SECONDARY TO AN ABSENCE OF OXYGEN

AMNIOTIC FLUID SERVES TO

INSULATE AND PROTECT THE FETUS

ANAPHYLAXIS IS CHARACTERIZED BY AIRWAY______ AND ____TENSION

SWELLING
HYPO

AREAS OF THE SPINE IN DESCENDING ORDER

-CERVICAL
-THORACIC
-LUMBAR
-SACRAL
-COCCYX

SMALLER VESSELS THAT CARRY BLOOD AWAY FROM THE HEART AND CONNECT THE ARTERIES TO THE CAPILLARIES

ARTERIOLES

AS A TRIAGE SUPERVISOR, YOU:

MUST NOT BEGIN TREATMENT UNTIL ALL PATIENTS HAVE BEEN TRIAGED

AS A WOMAN APPROACHES MENOPAUSE

HER MENSTRUAL PERIODS BECOME IRREGULAR AND VARY IN SEVERITY

AT THE ONSET OF AN ACUTE ASTHMA ATTACK, PATIENTS COMMONLY EXPERIENCE DIFFICULTY BREATHING AND

EXPIRATORY WHEEZING

NARROWING OF THE CORONARY ARTERIES DUE TO A BUILDUP OF FATTY DEPOSITS

ATHEROSCLEROSIS

AN INJURY THAT SEPARATES VARIOUS LAYERS OF SOFT TISSUE, RESULTING IN COMPLETE DETACHMENT OR A FLAP OF SKIN

AVULSION

THE AXIAL SKELETON IS COMPOSED OF THE...

-SKULL
-FACE
-THORAX
-VERTEBRAL COLUMN

BLEEDING FROM SOFT-TISSUE INJURIES TO THE FACE IS MOST EFFECTIVELY CONTROLLED WITH

DIRECT PRESSURE USING DRY, STERILE DRESSINGS

BRUISING TO THE RIGHT UPPER QUADRANT OF THE ABDOMEN FOLLOWING BLUNT TRAUMA IS MOST SUGGESTIVE OF INJURY TO

THE LIVER

CAPILLARY REFILL TIME IN AN INFANT

2 SECONDS

CARDIAC ARREST IN PEDIATRIC PATIENTS IS MOST COMMONLY THE RESULT OF

RESPIRATORY OR CIRCULATORY FAILURE

THE CERVICAL SPINE IS COMPOSED OF ___ VERTEBRAE

SEVEN (7)

IRREGULAR RESPIRATIONS CHARACTERIZED BY AN INCREASING RATE AND DEPTH OF BREATHING FOLLOWED BY PERIODS OF APNEA

CHEYNE-STOKES RESPIRATIONS

CIRCULAR MUSCULAR WALLS THAT REGULATE BLOOD FLOW THROUGH THE CAPILLARIES

CAPILLARY SPHINCTERS

COOL, CLAMMY SKIN
WEAKNESS
TACHYCARDIA
RAPID RESPIRATIONS ARE

CLASSIC SIGNS OF HYPOGLYCEMIA

IRREGULAR HEARTBEAT
SUDDEN UNEXPLAINED SWEATING
SHORTNESS OF BREATH / DYSPNEA

COMMON SIGN OF AMI

COMPARED TO AN ADULT, THE DIAPHRAM DICTATES THE AMOUNT OF AIR A CHILD INSPIRES BECAUSE

INTERCOSTAL MUSCLES ARE NOT WELL DEVELOPED

CONTRAINDICATION OF ADMINISTERING NITROGLYCERIN

SYSTOLIC BP IS LESS THAN 100MM HG

CONTRAINDICATIONS FOR ADMINISTERING ACTIVATED CHARCOAL

-INABILITY TO SWALLOW
-UNCONSCIOUS
-INGESTION OF ALKALI OR CAUSTIC MATERIAL

CORRECT DOSE OF EPINEPHRINE IN AN ADULR AUTO-INJECTOR

.3 MG

DEOXYGENATED BLOOD FROM THE BODY RETURNS TO THE

RIGHT ATRIUM

DETERMINING FREQUENCY OF LABOR CONTRACTIONS- TIME THE CONTRACTIONS FROM THE:

START OF ONE TO THE START OF THE NEXT

DIABETIC COMA

HYPERGLYCEMIA" COOL, CLAMMY SKIN

DISINFECTION IS MOST ACCURATELY DEFINED AS

KILLING PATHOGENIC AGENTS WITH A CHEMICAL MADE FOR THAT PURPOSE

DISLOCATION

DISRUPTION OF A JOING IN WHICH THE BONE ENDS ARE NO LONGER IN CONTACT

DISTRIBUTIVE SHOCK OCCURS WHEN

WIDESPREAD DILATION OF THE BLOOD VESSELS CAUSES BLOOD TO POOL IN THE VASCULAR BEDS

DOSE OF ACTIVATED CHARCOAL

1 GRAM FOR 1 KG OF BODYWEIGHT

DOSE OF ASPIRIN FOR CHEST PAIN

162-324 MG

DURING TRUE LABOE, UTERINE CONTRACTIONS....

BECOME MORE REGULAR

ECLAMPSIA

SEIZURES THAT RESULT FROM SEVERE HYPERTENSION

EPINEPHRINE IS INDICATED FOR PATIENTS WITH AN ALLERGIC REACTION WHEN _____ AND _____ ARE PRESENT

WHEEZING AND
HYPOTENSION

THE EXCHANGE OF O2 AND NUTRIENTS FOR WASTE PRODUCTS OF METABOLISM OCCURS AT THE CELLULAR LEVEL IN THE....

CAPILLARIES

EXTERNAL BLEEDING FROM A VEIN IS RELATIVELY EASY TO CONTROL BECAUSE

VEINS ARE UNDER A LOWER PRESSURE

THE EYEBALL ITSELF IS REFERRED TO AS

GLOBE

FUNCTION OF PLATELETS

FORMATION OF BLOOD CLOTS

GIVEN FOR CARDIAC-RELATED CHEST PAIN, __________ RELAXES THE WALLS OF THE CORONARY ARTERIES

NITROGLYCERIN

GOAL OF THE PRIMARY ASSESSMENT

IDENTIFY AND RAPIDLY TREAT ALL LIFE-THREATENING CONDITIONS

HEATSTROKE OCCURS WHEN

THE BODY'S HEAT-ELIMINATING MECHANISMS ARE OVERWHELMED AND CAN NO LONGER SWEAT

HEROIN IS AN EXAMPLE OF AN

OPIOID

HYPOTENSION IN A CHILD WITH BLUNT OR PENETRATING TRAUME OFTEN INDICATES:

THE LOSS OF HALF OF HIS/HER BLOOD VOLUME (THIS IS A LATE STAGE OF SHOCK)

HYPOTHERMIA OCCURS WHEN THE CORE BODY TEMP FALLS BELOW

95 F, 35 C

IN AN ADULT, BRADYCARDIA IS DEFINED AS

A PULSE RATE LESS THAN 60 BEATS/MIN

IN AN ADULT, TACHYCARDIA IS DEFINED AS

A HEART RATE GREATER THAN 100 BEATS/MIN

IN INFANTS AND CHILDREN, CAP REFILL GREATER THAN 2 SECS IS A SIGN OF

POOR PERIPHERAL PERFUSION

IN RESPONSIVE PATIENTS THAT ARE OLDER THAN 1, PALPATE THE PULSE AT THE _____ARTERY

RADIAL

INJURY TO A HOLLOW ORGAN WOULD MOST LIKELY RESULT IN

LEAKAGE OF CONTENTS INTO THE ABDOMINAL CAVITY

INSULIN SHOCK OCCURS IF

PATIENT TAKES TOO MUCH OF HIS PRESCRIBED INSULIN

ISCHEMIC HEART DISEASE

DECREASED BLOOD FLOW TO ONE OR MORE PORTIONS OF THE MYOCARDIUM

KUSSMAUL RESPIRATIONS ARE AN INDICATION THAT THE BODY IS..

ATTEMPTING TO ELIMINATE ACIDS FROM THE BODY

LEAF-SHAPED STRUCTURE LOCATED SUPERIOR TO THE LARYNX

EPIGLOTTIS

THE LEAST PRACTICAL PLACE TO STORE A PORTABLE OXYGEN CYLINDER IS:

IN THE DRIVER'S COMPARTMENT

THE MAJORITY OF A HORIZONTAL PATIENT'S WEIGHT IS IN THE

TORSO

THE MANNER IN WHICH THE EMT MUST ACT OR BEHAVE WHEN CARING FOR A PATIENT

STANDARD OF CARE

MAXIMUM DOSES FOR NITRO

3 DOSES

THE MOST APPROPRIATE LOCATION TO PARK YOUR AMBULANCE AT A HAZMAT INCIDENT IS:

UPWIND AT LEAST 100' AND UPHILL AT LEAST 25' FROM THE INCIDENT SITE

THE MOST SIGNIFICANT COMPLICATION ASSOCIATED WITH OROPHARYNGEAL SUCTIONING IS

HYPOXIA DUE TO PROLONGED SUCTION ATTEMPTS

THE MUSCULOSKELETAL SYSTEM

BONES AND VOLUNTARY MUSCLES

NARCAN IS ADMINISTERED BY THE EMT-B VIA:

AUTOMIZER

NEUROGENIC SHOCK OCCURS WHEN

FAILURE OF THE NERVOUS SYSTEM CAUSES WIDESPREAD VASODILATION

NITROGLYCERIN RELIEVES CARDIAC-RELATED CHEST PAIN BY

DILATING CORONARY ARTERIES AND IMPROVING CARDIAC BLOOD FLOW

NORMAL BLOOD GLUCOSE LEVEL

BETWEEN 80 AND 120 MG/DL

NORMAL RESPIRATORY RATE FOR AN ADULT

12 TO 20 BREATHS PER MINUTE

NORMAL SKIN COLOR, TEMP AND CONDITION

PINK, WARM AND DRY

OCCURS WHEN ORGANS PROTRUDE THROUGH AN OPEN WOUND

ABDOMINAL EVISCERATION

OPEN PNEUMOTHORAX OCCURS WHEN

AIR ENTERS THE PLEURAL SPACE FROM OUTSIDE THE BODY

OSTEOPOROSIS

DECREASE IN BONE MASS AND DENSITY

AN OXYGEN TANK SHOULD BE TAKEN OUT OF SERVICE AND REFILLED WHEN THE PRESSURE INSIDE IS LESS THAN

LESS THAN 500 PSI

A PATIENT WHO IS SWEATING PROFUSELY IS REFERRED TO AS BEING

DIAPHORETIC

PATIENTS WITH RIB FRACTURES WILL COMMONLY (BREATHE)

RAPIDLY AND SHALLOWLY

PEDIATRIC DOSE OF EPINEPHRINE

.15 MG

PERFUSTION

CIRCULATION OF BLOOD WITHIN AN ORGAN IN ADEQUATE AMOUNTS TO MEET THE BODY'S METABOLIC NEEDS

PERITONITIS USUALLY OCCURS WHEN

HOLLOW ABDOMINAL ORGANS SPILL THEIR CONTENTS

PID DOES NOT TYPICALLY AFFECT THE

URINARY BLADDER

THE PRIMARY FUNCTION OF THE RIGHT ATRIUM IS TO...

RECEIVE BLOOD FROM THE VENA CAVA

PLACENTA

ALLOWS 02, CO2, AND OTHER PRODUCTS TO TRANSFER BETWEEN MOTHER AND FETUS BUT DOES NOT ALLOW THEIR BLOOD TO MIX

POSITIVE INDICATORS OF A FRACTURE

CREPITUS AND FALSE MOTION

PULMONARY EDEMA AND IMPAIRED VENTILATION OCCUR DURING WHICH TYPE OF SHOCK?

CARDIOGENIC SHOCK

QUID PRO QUO

TYPE OF SEXUAL HARASSMENT, WHEN HARASSER REQUEST SEXUAL FAVORS IN EXCHANGE FOR SOMETHING ELSE

SHOCK IS THE RESULT OF

HYPOPERFUSION TO THE CELLS OF THE BODY

SKELETAL MUSCLE IS ALSO CALLED

VOLUNTARY MUSCLE

SOLID ABDOMINAL ORGANS INCLUDE THE

SPLEEN
KIDNEYS
PANCREAS

THE SPINAL CORD EXITS THE CRANIUM THROUGH THE

FORAMEN MAGNUM

SPLEEN

ASSISTS IN BLOOD FILTRATION, SERVES AS A BLOOD RESERVOIR, AND PRODUCES ANTIBODIES

STRIDOR

HARSH, HIGH-PITCHED INSPIRATORY SOUNDS IN THE UPPER AIRWAY

SUCCESSFUL TREATMENT OF A STROKE DEPENDS ON:

WHETHER THROMBOLYTIC THERAPY IS GIVEN WITHIN 3 HOURS AFTER SYMPTOM ONSET

SUPINE HYPOTENSIVE SYNDROME

THE PREGNANT UTERUS COMPRESSES THE INFERIOR VENA CAVA

SYNCOPE IN OLDER PATIENTS IS CAUSED BY

INTERRUPTION OF BLOOD FLOW TO THE BRAIN

SYSTOLIC PRESSURE

THE PRESSURE EXERTED AGAINST THE WALLS OF THE ARTERY WHEN THE LEFT VENTRICLE CONTRACTS

THREE MAJOR PARTS OF THE BRAIN

CEREBRUM
CEREBELLUM
BRAIN STEM

TREATMENT FOR JELLYFISH STING

IRRIGATING WOUND WITH VINEGAR AND IMMERSING HIS LEG IN HOT WATER

A TUBE FROM THE BRAIN TO THE ABDOMEN THAT DRAINS EXCESSIVE CS FLUID

SHUNT

TWO MOST COMMON SIGNS OF ANAPHYLAXIS

WHEEZING
URTICARIA (HIVES)

TWO WAYS OUR BODY ELIMINATES HEAT

SWEATING
DILATION OF SKIN BLOOD VESSELS

TYPE 1 DIABETES

CONDITION IN WHICH NO INSULIN IS PRODUCED BY THE BODY

USUAL DOSE OF ACTIVATED CHARCOAL MAXIMUM ADULTS AND PEDIATRIC

PEDIATRIC- UP TO 25 GRAMS
ADULT- UP TO 50 GRAMS

THE VOCAL CORDS ARE LOCATED IN THE

LARYNX

WHAT ACTIVITIES OCCUR IN THE "WARM ZONE"?

DECONTAMINATION

WHAT CAN CAUSE THE PUPILS TO REMAIN SIGNIFICANTLY CONSTRICTED?

OVERDOSE OF AN OPIATE DRUG

WHAT IS THE TYPICAL FUNCTION OF THE RESCUE TEAM?

TO PROVIDE SAFE ENTRY AND ACCESS TO PATIENTS

WHAT PHYSIOLOGIC ACTIONS DOES EPINEPHRINE PRODUCE WHEN GIVEN FOR AN ALLERGIC REACTION?

VASOCONSTRICTION
BRONCHODILATION

WHAT RESPONSIBILITY/DUTY DOES NOT FALL WITHIN THE REALM OF THE MEDICAL BRANCH OF THE ICS?

EXTRICATION

WHICH COMPONENTS ARE NEEDED TO PROVE NEGLIGENCE?

-DUTY TO ACT
-BREACH OF DUTY
-INJURY/DAMAGES
-CAUSATION

WHICH OF THE FOLLOWING MEDICATION ROUTES HAS THE SLOWEST RATE OF ABSORPTION?

ORAL

abdominal thrusts

A four-year-old male patient has just swallowed a marble in front of you, which has caused a complete (severe) airway obstruction. Your immediate care should include:

pulling the object out in the direction it entered the cheek

Care for the patient with an impaled object in the cheek includes:

sucking chest wound

An open wound to the chest that allows air to pass through it, causing shortness of breath and bubbling of blood around the wound, is called a:

immobilizing the injury site and the joints above and below it

4. General rules of immobilization of extremity injuries include:

capillary refill

One of the most reliable signs of decreased perfusion in the pediatric patient is

The cause of febrile seizures in children is:

a rapid rise in temperature

Your unit has arrived on the scene of a private residence where the scene is determined to be safe. According to the parent who meets you at the door her six-year-old son may have taken a handful of pills, thinking they were candy. To treat the child who

activated charcoal

Which of the following patient care equipment should be carried in various sizes (such as pediatric, adult, etc.)?

bag mask devices

The ambulance vehicle should be regularly inspected. Inspections done while the ambulance engine is turned off include

the level of coolant/antifreeze and the pressure cap indicator

Most states allow ambulances responding to emergencies with lights and sirens to disregard certain traffic laws. The emergency vehicle operator

can exceed the posted speed limit as long as life or property is not endangered.

Ground gradient is electricity spreading out from a power line touching the ground. If you feel a tingling sensation in your legs you should

hop to a safe place on one foot.

Information you should report back to dispatch about the scene size-up of an MCI includes all of the following, except

the destination hospital of all patients

The area of the spine that is the least susceptible to injury would be the _________ spine

thoracic

The nerves that control the involuntary functions of digestion, the bowels, and the bladder are part of the _____________ nervous system

autonomic

You are on the scene of a 58-year-old male patient who collapsed in the bathroom at a local resturant. Bystanders determined he had no signs of circulation so they started CPR. After delivering your first shock with an AED, if the patient is still pulsele

perform two minutes of CPR and reassess

The four types of poison classifications include ____ poisons

inhaled

Modern treatment of poisonings and overdoses consists primarily of all the following, except

administration of an antidote

The exchange area where oxygen and nutrients from the mother's bloodstream are carried across to the fetus is called the

placenta

Delivery of the baby may be imminent when the mother's contractions:

last 30 seconds to 1 minute, and are 2 to 3 minutes apart

A prolapsed umbilical cord, when the cord is pinched between the vaginal wall and presenting part of the baby, is handled by

inserting several fingers into the mother's vagina and gently pushing the presenting part off the cord.

Which of the following is true regarding miscarriages and abortions?

The miscarriage is commonly considered spontaneous, and the abortion induced

Of the three major types of blood vessels, the vessels that carry blood away from the heart are called

arteries

The abbreviation NKDA means

no known drug allergies

The abbreviation STAT means

immediately

The main components of an adult's vital signs include assessment of breathing, skin, pupils, blood pressure, and

pulse

Which artery should be checked first by a healthcare provider for a pulse in the unresponsive infant?

Brachial

The skin should be assessed for

color, temperature, and condition.

In a dark room, the normal pupil should

dilate

What is body mechanics?

Methods developed for the proper positioning of the patient during lifting and moving operations

Which of the following moves is not considered an emergency move?

rapid extrication

The vocal cords are found in the

larynx

Your unit is on the scene of a 28-year-old male who was found by a coworker unconscious at his desk. Once you assure the scene is safe you begin your initial assessment. Which airway maneuver should be used to open the airway of an adult without suspected

head tilt/chin lift

You are treating a 39-year-old female who had an apparent seizure. She has a lot of secretions and foam around her mouth and nose, so you decide to suction her. What body substance isolation precautions should be used during suctioning?

gloves, mask, and eye protection

Irritability with co-workers, inability to concentrate, indecisiveness, loss of appetite, or isolation, are all warning signs of:

stress

One of the best techniques to use when working with death and dying is to just

listen

A mask should be worn by the EMT-Basic to prevent the spread of disease

during bleeding control

When delivering artificial ventilations to an adult patient using a pocket mask, it is important to

take a deep breath and slowly ventilate over 1 1/2 to 2 seconds

The preferred method of inserting an oropharyngeal airway in a child is

using a tongue depressor to press the tongue down while inserting the airway.

When your unconscious patient needs to be suctioned, due to a collection of secretions that are accumulating in his mouth, you should utilize a rigid tip (Yankauer) whenever possible. If you only have a flexible catheter to suction with remember to

never lose sight of the tip

Your unit is dispatched to a call for a victim of an assault in a local bar. As you call for the police to respond, you review in your mind the steps of the scene size-up. All the following are elements of a scene size-up, except

patient transport priority decision

The patient carrying device most commonly used for removing patients from vehicles is the

KED

In the term SAMPLE history, each letter stands for an element of the history. Which of the following is NOT one of those elements?

Length of illness

Blood in the ventricle is prevented from being forced back up into the atrium when the ventricle contracts by:

a one-way valve between the atrium and the ventricle.

Your unit has just arrived on the scene of a 34-year-old male patient who was found by a co-worker at the base of the stairwell. He states the patient has not been responding for the past 20 minutes. You assume that an unconscious medical patient would gi

implied

Guidelines that define the extent and limits of the job the EMT-Basic does are referred to as:

the scope of practice

The elbow is____to the shoulder since it is farther from the torso than the shoulder

distal

Forcing a competent patient to go to the hospital against his will may result in being charged with

assault and battery

Your unit is on the scene of a 68-year-old male patient who is complaining of a tearing sensation in his abdomen. He has a history of hypertension and you find there is a mass in his abdomen, which has a pulse. A weakened section of an arterial wall that

aneurysm

A patient with cardiac compromise should be transported immediately if he has:

no history of cardiac problems

Common symptoms of a patient in cardiac compromise include

difficulty breathing

A diastolic blood pressure greater than 90 mmHg is considered:

hypertension

Some patients complain of irregular or rapid heartbeats that they can feel as a fluttering sensation in the chest. This is called:

palpitations

When an asthma attack occurs:

stale air becomes trapped in the lungs

An infant or child patient who exhibits wheezing, increases breathing effort on exhalation, or rapid breathing without stridor, probably has a(n):

lower airway disease

You obviously will not be able to obtain a SAMPLE history from the patient if he is unresponsive. However, you may be able to obtain one from

family,friends,co-workers

Seesaw" breathing is:

a sign of inadequate breathing

Agonal respirations are:

shallow and gasping.

Upon arrival at the scene of a 68-year-old man who was having chest pain, the patient's wife tells you he just took a nitro tablet and has experienced some relief. Nitroglycerin tablets are administered:

sublingually

The EMT-Basic may assist a patient in using epinephrine when administered:

by auto-injector

Your unit has arrived on the scene of a 19-year-old female who was painting the deck when she was stung by a group of bees. She states she is highly allergic to bee stings. The medication used to reverse a life-threatening allergic reaction by constrictin

epinephrine

If your patient has an altered mental status and a history of diabetes, you may give him:

oral glucose

To the general public, the word "drugs" generally refers to:

illegal or abused substances

The study of drugs, their sources, characteristics, and effects, is called:

pharmacology

Which of the following is true regarding the prehospital care report?

It becomes part of the patient's permanent hospital record.

Patients who are hearing impaired or deaf

can often read your lips.

The agency that assigns and licenses radio frequencies is the:

Federal Communications Commission.

The memory aid OPQRST includes letters standing for:

severity" or how bad is the pain?

A significant mechanism of injury for a child is a fall:

from more than 10 feet

Techniques for applying a cervical collar include:

keeping the patient's hair out of the way.

The memory aid DCAP-BTLS includes which of the following?

Abrasions

You are on the scene of a 29-year-old female who was painting and fell off a ladder onto her head. During your initial assessment she responds only when you pinch her toe or rub her sternum briskly. This level of responsiveness would be:

painful

Based on your immediate assessment of the environment and the patient's chief complaint and appearance, you form a(n):

general impression

Because of multiple impacts, the type of collision that is potentially the most serious is the:

rollover

Signs and symptoms of shock (hypoperfusion), which are actually the body's compensating mechanisms, include all of the following except:

increased urine output

Your unit is on the scene of a 18-year-old woman in active labor. She states that her water broke, and lets you check for crowning. You note that the umbilical cord is sticking out of her vagina. How should the EMT-Basic transport a patient who presents w

in a knee-chest (hip elevated) position

Of each of the following methods of bleeding control, which should be used first?

direct pressure

Signs of blunt trauma include all of the following, except:

deep, sharp cuts

The functions of the integumentary system are: protection, sensation, excretion, vitamin D synthesis, and

temperature

There is a potential for internal bleeding or life-threatening injuries to internal organs from a(n):

puncture

Your unit is standing by at the scene of a residential fire. Suddenly a firefighter comes out carrying a 48-year-old female who is severely burned on the upper half of her body. Burn severity depends on each of the following factors, except

the type of object that was burning

A patient has sustained a third-degree burn on the back of her torso and the back of her left arm. What percentage of her body surface area is burned?

22.5

The key functions of the musculoskeletal system include:

support, protection, movement

To select the proper size oropharyngeal airway, you should measure from the patient's:

corner of the mouth to the tip of the earlobe

The technique for inserting a nasopharyngeal airway includes:

gently pushing tip of the nose upward

The yoke of a portable oxygen pressure regulator is designed so it will fit an oxygen tank and not on a tank containing another type of gas. The system used to assure this is called:

the pin-index safety system

A patient who is said to be in respiratory arrest has:

no breathing but may still have a pulse

A bag mask should be connected to an oxygen inlet flow of _____ liters per minute

15

When managing the airway of a pediatric patient, the EMT-Basic should consider:

the mouth and nose are more easily obstructed than in adults.

The reason a patient calls EMS is referred to as the:

chief complaint

In unresponsive patients the chief complaint may be determined by all of the following except:

the patient

Capillary refill is most reliable when assessed:

at room temperature

The main purpose for checking circulation during the initial assessment is to:

determine if the patient has a pulse

To complete the initial assessment the EMT-Basic should:

make a priority transportation decision.

You have just completed the initial assessment of a 30-year-old victim of a fall. According to the patient he is not sure if he lost consciousness. The first step in the focused history and physical exam of the trauma patient is to:

perform the rapid trauma exam

Assessment of ________ should be done before and after immobilization of any extremity

proximal pulse, motor function, and sensation

The ongoing assessment is performed on:

all patients who are transported

Trending a patient's condition is something that is done in the

ongoing assessment

How often should you reassess an unconscious medical patient who has a nasal airway and nonrebreathing mask in place?

Every 5 minutes

A medication, or drug, can have up to _____ different names.

4

A non-desired medication reaction that occurs, in addition to the desired reaction, is known as a:

side effect

A(n) _____ is a form of medication with drug particles that are mixed in a suitable liquid.

suspension

Documentation of medication the EMT-Basic administers should include

improvements in a patient's condition, route and time of administration, deterioration of a patient's condition.

Your unit is on the scene of a 38-year-old female who may be having an asthma attack. As you enter the room you notice audible wheezing and the patient is sitting with her elbows out. This tripod position is associated with a patient having:

severe respiratory distress

The EMT-Basic can provide positive pressure ventilation to a patient using a(n):

bag-valve mask

A type of medication designed to relax and open the smaller airways, helping to improve breathing, is called a(n):

a bronchodilator

A 32-year-old female has a complaint of sudden onset of dyspnea with no history of any respiratory problems. She is a one-pack-a-day smoker, takes an oral contraceptive but no other meds and has no allergies. Which of the following conditions should be hi

pulmonary embolism

Pneumonia is primarily an acute infectious disease and usually presents with a fever in patients, except:

the elderly

A(n) ______ will increase the work of breathing and the body's oxygen demand, especially in children.

increased stress level

Seesaw or rocky breathing, exhibited by the chest drawn inward and the abdomen moving outward, is a sign of:

extreme inspiratory efforts in children

Your unit is on the scene of a 58-year-old male patient who has had crushing substernal chest pain for the past 30 minutes. You have taken a set of vitals and are administering oxygen. When assessing and treating a patient with chest pain or discomfort, t

not take the time to diagnose what type of cardiac emergency is occurring.

Anatomical differences between adults and children include:

the infant's head is proportionately larger than the adult's.

Your treatment for a 3-year-old male in severe respiratory distress should include:

assisted ventilations, using a bag-valve mask and oxygen

In infants and children, the primary cause of nontraumatic cardiac arrest is

respiratory failure

You are treating a 9-month-old who "fell out of the crib while sleeping." You note the child is only responsive to painful stimuli, and note the soft spots on the skull (fontanelles) look swollen. What should you suspect?

shaking baby syndrome

In some pulmonary diseases, such as emphysema, the pulmonary vessels are compressed, making it much more difficult for the _____ to pump the blood out.

right ventricle

The AED is intended for use with:

all cardiac arrest patients with shockable rhythms, except infants

The heart's typical response to ischemia is

chest discomfort

Using the AED, the EMT-Basic delivers one shock to an apneic and pulseless patient. The AED now states, "no shock advised." What most likely is the reason for this alert from the AED?

The patient is no longer in V-fib

Your unit is on the scene of a 69-year-old male whose wife called the ambulance because she was concerned that his color was not good and he has a cardiac history. His vitals are respirations of 20, pulse of 88 and BP of 138/82. Which of the following is

generalized weakness

The two primary hormones that are responsible for controlling the blood sugar level are ____ and ____, both of which are secreted by the pancreas.

insulin:glucagon

The EMT-Basic arrives at an office to find multiple patients with altered mental status. The EMT-Basic should suspect _____ is causing the illness.

some type of hazardous gas or poison

Your unit is on the scene with a 18-year-old patient who suddenly became ill after eating in a seafood restaurant. His color is pale and his vital signs are: respirations of 28, pulse of 120 and BP of 110/60. You suspect an allergic reaction is the proble

medications that are taken orally or by injection

The most significant mechanism of heat loss is

radiation

The first step in the care of a patient presenting with confusion, who is found in a hot environment, is to:

move the patient to a cool environment as quickly as possible.

A patient with a core body temperature of ______ is considered to be hypothermic.

95F or less (35C or less)

All of the following are potential hazards of improper splinting, except:

reduction of pain

A(n) ______ is the most important sign in cases of suspected head injury.

creasing mental status

Which of the following statements about skull injuries is most accurate?

The deformity itself does not cause disability or death.

When a patient with a suspected spine injury must be moved before being immobilized, this is called

rapid extrication

A tool for assessing a patient's level of responsiveness that uses numerical values to score responses is:

the Glasgow Coma Scale

Paralysis of the respiratory muscles may occur with injury to the _______ spine.

cervical

A sign of severe head injury in which the systolic blood pressure increases, the heart rate decreases, and the respiratory pattern changes, is known as:

Cushing's reflex

Medical factors that could mimic a behavioral emergency include:

excessive cold, hypoxia, infections

Most ____ patients have elaborate delusions, mostly of persecution.

paranoid

Which of the following statements about behavioral emergencies is most accurate?

Not all behaviorally disturbed patients will become violent.

Signs and symptoms of a possible spontaneous abortion include all of the following, except:

crowning

You have just assisted a mother with the delivery of a healthy full-term baby girl. The apgar is a 10 and the child is resting. You notice that the mother continues to bleed at a rapid pace, so you attempt uterine massage. Uterine massage helps to stop va

stimulating contractions, which decreases the uterine size

During the transportation of a woman in her third trimester, the EMT-Basic should place the patient in a sitting position to avoid

supine hypotension syndrome

You have just arrived on the scene of a woman who is in active labor. Her water broke already and it appears from your inspection for crowning that birth is imminent. In a normal delivery the head will deliver in a ____ position and turn:

face down / sideways

Fluids should be removed from the newborn infant's airway

as soon as the baby is delivered.

Your unit has just arrived on the scene of a two-vehicle head-on collision. One patient is dead at the scene and the other driver is entrapped in his vehicle. To gain access to a patient trapped in a vehicle the EMT-Basic should

break the window furthest from the patient.

When responding to a call, minimal lighting should be used

during heavy fog

Your unit is arriving at the scene of an overturned tanker truck. After calling for additional help and identifying the substance from the placard on the truck, you establish a location to treat the exposed patients. The _____ zone at the site of a HAZ-MA

cold

Which of the following is least accurate about the chest of a child? The rib cage is more flexible

decreasing the likelihood of internal organ damage with blunt chest trauma.

In children _____ provide more valuable information about the perfusion and shock status than the blood pressure

skin color, temperature, and condition

a tort is a

civil wrong

When the body's immune system detects an antigen, the response is to:

produce antibodies

The EMT-Basic may be required to make a decision about any of the following aspects of care, except:

when to access the public service answering point (PSAP).

A woman in labor states that her water broke, the fluid is stained green. This is called _______ staining and is an indication that the baby is _________.

meconium: stressed

What is the organ that contains the developing fetus?

uterus

What is the outgrowth of the developing fetus that attaches to the wall of the uterus and supplies the fetus with nutrients called?

placenta

What do you do with about the placenta once you have successfully delivered the baby and cut the cord?

initiate transport and allow the placenta to deliver spontaneous.

What is it called when during your examination, during a contraction, the top of the head of the baby appears at the vaginal opening called?

crowning

What is the best course of action when you are on scene with a 25 y/o female who is 37 weeks pregnant and she states she started bleeding bright red blood but is not having contractions?

immediate transport

How is the period of pregnancy from conception to birth divided?

trimesters

What should you do if you find the cord wrapped around the neck, when the baby is delivered?

pull cord over baby's head if possible

What is contraindicated when a 26 y/o woman, who is 38 weeks pregnant, calls for help because "something fell out". On examination you find the umbilical cord hanging out of vagina.

with a gloved hand, push umbilical cord back into vagina

When does the third stage of labor end?

placenta is delivered

What should you do once the baby has been delivered and has an apgar of 3 at one minute and after stimulation, you note that the pulse rate is 50 and the baby is not breathing.

start compressions and BVM with 100% oxygen

What may be imminent of delivery of a baby?

1. contractions are 2-3 mins apart
2. mother has urge to push
3. contractions last 3-4 mins
4. bag of waters break
5. head is crowning

What is called when you are delivering a baby when you become aware that the presenting part is the buttocks.

Frank breech

What fluid is the fetus floating in the "bag of waters"?

amniotic

What is the first thing you should do when you have delivered an infant who is limp and blue?

1. suction
2. dry
3. stimulate

What is not appropriate when you have successfully delivered a baby and the placenta and you have noted that there are still vaginal bleeding?

pack vagina with sterile pads

What should you do first when you are transporting a 32 week pregnant woman who was involved in a motor vehicle crash and she is immobilized on a back board, while en route, you note that her blood pressure has dropped to 85/40?

tilt backboard up to the left side

What is the APGAR score 5 mins after delivering a baby and you note that the baby has blue extremities, a pulse of 120, little response to suctioning, vigorous activity and good respirations with a strong cry?

8

What are the signs of APGAR?

1. appearance
2. pulse
3. grimace
4. activity
5. respirations

What might you have to do aftr the baby's head has delivered?

1. Suction mouth then nose
2. wipe mucous from baby's mouth

What should be your first action when the 3 y/o child is in severe respiratory distress from asthma and you note rapidly level of consciousness and a pulse of 40?

BVM with 100% oxygen

What should you with the parent or caregiver when caring for a sick child?

assist with the care of the child as much as possible

What should you do if you are caring for a 2 y/o child who fell down stairs and injured his left arm and during your exam, you notice bruises to chest and abdomen and what appears to be cigarette burns on back?

report findings to ED, police and social services

What findings would lead you to believe a child is dehydrated when you are examining a child's fontanelle?

sunken and non-pulsatile fontanel

What is the best course of action when you have responded to the scene of a child not breathing and he was put down for a nap and found apneic 2 hrs later, which you find the child still warm to the touch?

start cpr and immediately transport

What does the pediatric airway have in comparison to an adult airway?

proportionally larger tongue

Why do children develop hypothermia more easily than adults?

larger surface area in proportion to body mass

What may stridor be a sign of?

1. epiglottitis
2. foreign body
3. croup

What would be the first step when you find an unresponsive child victim who has agonal gasps and someone else is calling to activate EMS?

start CPR

What is a child in who exhibits increased work in breathing?

respiratory distress

What should you do if you are trying to examine a 6 y/o child with abdominal pain and she is being very uncooperative.

allow mother to hold her while examine her as best as you

What is important to remember when dealing with a toddler?

they do not like to be touched by strangers

What rate should you ventilate at when ventilating a child in respiratory failure?

12-20

What is the most common cause of cardiac arrest in children?

respiratory failure

What should you do if you have a 6 y/o patient at a soccer match that has collapsed and adequate CPR was initiated prior to your arrival?

immediately place AED pads and analyze the rhythm

What must be done if a procedure must be done that may cause pain?

tell child it may hurt

What is an accurate indicator of early hypoperfusion in children?

delayed capillary refill

What must be done when opening the airway of an infant or young child?

extend neck sufficiently to open airway

What should be done when caring for a sick or injured child?

allow parent to hold child when possible

Within the first minute of observing a sick or injured child what should you base your initial treatment on?

1. skin color
2. work of breathing
3. general impression

What accurately describes the changes in the body that occur with aging?

muscle strength decreases and is not easily built up

What should the EMT-B do when obtaining a history from an elderly patient?

address patient as mr or mrs unless asked otherwise

what is contraindicated when you are assessing a 76 y/o female with an injured arm when she has obvious hearing and visual deficits as you try to communicate?

shout loudly

What is the best place to check for signs of dehydration?

mucous membranes

What should you do when assessing an 82 y/o male who is confused and disoriented?

check with family to determine if this is normal for the patient

What is the respiratory rates for 0-1 yoa?

30-60

What is the respiratory rate for child?

15-30

What is the respiratory rate for teens?

12-20

What is the heart rate for 0-1 yoa?

100-200

What is the heart rate for a child?

60-140

What is the heart rate for teens?

60-100

Under what conditions is it appropriate to use a police or other emergency vehicle escort?

as a last resort

Under what condition should an EMT exceed teh posted speed limit?

situation is critical

What 4 times should you notify dispatch?

1. leaves the scene 2. arrives at hospital 3. back in service 4. arrives at scene

What is your first responsibility once you transferred care of your patient?

write PCR

What is the EMT's first duty to the patient while arriving at scene?

safety

What is the symbol of EMS called?

star of life

What factor concerning your chosen landing site would make it useable by helicopter, once you have requested a helicopter response?

100 x 100 ft in area

What should be used to clean an ambulance when there is no visible blood or body fluid contamination?

EPA approved germocide

what part of the ambulance does not require a check at the beginning of each shift?

tie rod ends

Under what circumstances may an EMT by-pass driving laws?

when running as an EMS provider on EMS call

What direction should an EMT approach a vehicle that has been involved in a collision and has a patient inside?

front

What is the easiest way to eliminate electrical current and associated hazards in vehicle collisions?

turn off engine

What is the best way to gain quick access to a patient who is still in a vehicle following a crash?

check all doors

What is the safest way for an EMT to approach a scene with downed wires?

assume wires are live and do not approach

What is the best method for protecting a patient during extraction from a vehicle?

cover patient and EMT-basic with heavy fire proof covering

Where should you park your ambulance if you are the first responder to arrive at the scene of a motor vehicle accident?

on shoulder, between traffic coming up behind you and the crash scene

Where should you park ambulance when you recognize that a truck has rolled over and is leaking liquid?

uphill and downwind

What is the role of the EMT in vehicle stabilization and patient extraction?

patient care provider

What is the first thing an EMT should do when arriving at the scene of a motor vehicle crash?

make sure the scene is safe

What situation is most likely to involve simple access?

residential home

In what zone are normal triage, stabilization and treatment performed?

cold

What is the first and primary concern of the EMT at a hazardous material incident?

personal safety

What would a number 3 in the blue area indicate?

extreme danger to health

What is not a potential danger posed by hazardous materials?

infection

What is the minimal level of training recommended by OSHA for EMS providers likely to witness or discover a hazardous materials emergency?

first responder awareness

In which zone is initial (gross) decontamination performed?

hot

What type of hazardous material does a red placard with a flame symbol?

flammable gas or liquid

What is the definition of a hazardous material?

unreasonable risk or unreasonable risk to health

What information form is the workplace required to have available if you respond to the scene of an injury from a chemical spill?

msds

in what zone are life saving procedures such as airway management and immobilization performed?

warm

What activity is the staging sector responsible?

controlling ambulance flow

What basis should patients be moved from triage sector to treatment sector?

priority

What would you do if you are an EMT working a mass casualty scene?

assist at local hospitals if requested

What color are delayed care patients in a standard 4 level triage system?

yellow

Who should be assigned as the triage officer?

most qualified emt

What is a set of signs and symptoms would indicate a red tag description?

20% burns with stridor and wheezing

What would be the level of a patient who has a life-threatening injury, but is breathing and has a pulse?

immediate

Who should be responsible for assuming command of a multiple casualty incident when the first units arrive?

senior emt at scene

What is the sector responsible for distributing medical material and equipment called?

supply

What phase of an explosion causes damage through a pressure wave?

primary

What are 4 characteristics of vesicants?

1. treated by immediate irrigation 2. phosgene and mustard gas 3. blistering agents 4. delayed effect

Under what circumstances does a patient represent a potential danger to others when he presents with possible radiation exposure?

radioactive material in wounds

What types of injury are you anticipating to treat when you are responding to WMD scene where an incendiary device has been used?

heat burns

What should you do first when your unit is the first to arrive on scene at a possible WMD incident?

incident command system

What are biological weapons?

infectious agents

In what settings besides a terrorist attack might you encounter a " nerve agent" ?

1. household 2. industrial 3. agricultural

What type of chemical agent is most likely to have been exposed when your patient has been exposed to an unknown chemical agent?

vesicant

In which settings would there most likely be an adequate response to a WMD attack?
a. large metro area
b. suburban community
c. rural community

none

What does "SLUDGE" refer to?

effects of organophosphates

What are the 4 most likely injuries you will find on the patient closest to the impact when a car is t boned?

1. head 2. upper spine 3. pelvis 4. long bones

How is the pattern of injury in an auto-pedisterian collision different between adult and child?

child will turn towards vehicle

What is the most improtant factor in determining the amount of kinetic energy of a moving object?

velocity

What other area should be examined if a 23 y/o female falls from s second story window, landing on her feet?

upper lumbar spine

What is most likely to occur when a person is holding their breath when his chest impacts against a solid object?

alveoli may rupture, called a "paper bag" injury

What is it called when a bullet creates a path through a soild organ that is wider than the bullet itself, thereby increasing the damage?

cavitation

What is a priority action that should occur on scene when you have established that your patient has a high risk mechanism of injury and is a priority trauma patient?

spinal immobilization

What 3 impacts are involved when a car runs into a telephone pole?

1. organs against interior of body 2. auto into pole 3. driver into car interior

What 5 aspects are a potential type of damage from a blast injury?

1. body striking another object 2. penetrating wounds 3. fractured limbs 4. internal organs striking inside body 4. shock wave damage

What 3 organs may be possibly injured if your patient was stabbed in an upward direction by a 6 inch blade just under the costal margin on the right side of his body?

1. liver 2. lung 3. bowel

What would be a possible complication if your patient was stabbed with a 6 inch knife in the anterior part of the chest?

1. infect 2. bleeding in the chest

What is the most appropriate management of the avulsed segment of tissue if your patient has suffered a skin avulsion of the upper arm that is still attached to the body by a small piece of skin?

place skin over wound and dress with sterile dressing

What si an occlusive dressing?

does not allow air to penetrate

A tongue caught in the mouth of a beer bottle is an example of clamping injury? T or F

true

What would you most likely expect if your motorcycle rider "laid the bike down" and was not wearing protective gear?

multiple abrasions

What 3 things may a crush injury cause?

1. substantial bleeding 2. self healing 3. cause cardiac arrest

What action is most appropriate if your patient has a hematoma and swelling along teh middle of her left forearm following an assualt with a baseball bat?

splinting for possible fracture

What is a hematoma?

large collection of blood under the skin

What is an avulsion?

flap of skin that has been torn off underlying tissues

What is a puncture?

wound that is deeper than it is wide

What is abrasion?

part or all of the epidermis is scraped off

What is a contusion?

discoloration and swelling within the dermis

What is a laceration?

linear cut or tear through the skin

How do you treat a patient that has a small laceration below his right eye which is bleeding profusely?

pressure dressing

What is the most appropriate treatment for the stump wound with a patient with an amputated thumb?

indirect pressure

What is the most appropriate action if you have applied an occlusive dressing to a sucking chest wound and after this your patient complains of increasing difficulty breathing and his BP starts to drop?

remove dressing and then reapply

Which statement is most true of bandaging and dressing a wound?

a dressing should be sterile and held in place by the which need not to be sterile

What is the 3 things true of amputations?

1. may not bleed 2. amputated part must be found an dkept cool 3. trasnported to specialty facility

What is the 6 classic sequences in controlling teh bleeding if your patient has spurting, bright red blood coming from a laceration on his leg?

1. direct pressure 2. pressure dressing 3. additional guaze pads 4. elevate extremity 5. apply pressure to pressure point 6. tourniquet

What is the best method for handling an impaled object in the cheek or mouth that is not and will not obstruct the airway?

leave in place and stabilize with bulky dressing

How do you treat an injury if you had a patient running with a screw driver in his hand and tripped over his dog and the screwdriver went into his chest just below the clavical?

stabilize the screwdriver with a bulky dressing and be prepared to treat the patient for shock

What is the most correct treatment for this patient that you and your partner respond to an MVC and the driver appears unresponsive and you see major bleeding across his lower abdomen. upon inspection you find a small area of bowel protruding through teh

apply a sterile moist dressing, then an occlusive dressing and be prepared to treat for shock

What is your primary concern for this baby if you are called for a baby whose big brother pushed her down and she bit her tongue and it is bleeding profusley?

airway

What is the layer of skin that contains nerves, blood vessels, sweat and subceous glands and hair follicles?

dermis

What are burns that are characterized by charred or leathery consistency with no pain sensation called?

full thickness

What is the estimated burn percentage if an adult has suffered a partial thickness-burn to the anterior portions of both arms, chest and abdomen, using the rule of nines?

27

What is a burn that goes completely around a limb or torso called?

circumferential

What kind of burns are large fluid filled blisters?

partial thickness

Which is the most appropriate first step in the mangement of a skin exposure to a dry cuastic chemical?

brush of chemical

What is the first step in caring for a patient that has suffered an electrical burn to the leg after falling on a downed power line?

confirm poer is off

Which should be removed from a burn patient?

rings and jewelry

What is the most appropriate first treatment for an injury of battery acid has splashed into both eyes of the patient?

irrigate copiously with water

What makes the circumferential burns so dangerous besides the burn itself?

possibility of compartment syndrome

What is teh severity of a partial thickness burn encompassing 22% of the BSA of a child less than age 5?

critical

What is the tough fibrous tissue that connects muscles to bones?

tendon

What will you do if you have applied the PASG to a suspected femur fracture: you are at 11,000 feet and your ED is at 5,000 feet?

increase pressure in PASG

What must you do to correctly splint a long bone?

immobilize joints above and below injured bone

What is the bone of the upper extremity?

humerus

What is the most appropriate action when you are 30 mins from a hospital from the hospital with a patient who has a midshaft injury of the humerus and there are no distal pulses palpable in the left arm?

contact medical control and obtain permission to manupulate the injury

What is your first step in teh treatment if your patient has suffered a fracture of the lower leg with the ends of the bones exposed and the patient is talking to you and crying.

cover wound with sterile guaze

What splint is best suited for a shoulder fracture or dislocation?

sling and swathe

What is teh most appropriate splinting technique for your patient who has suffered an open femur fracture 3 inches above the knee?

traction splint

What effects does applying ice to an injured extremity accomplish?

vasoconstriction and decrease swelling

What is this type force when a patient falls on an outstretched hand and fractures the radius at the elbow?

indirect

What will your next action be when you have just applied a long bone splint to a suspected tibia fracture when you reassess PMS and you find that the pulse is no longer palpable?

lossen splint and recheck PMS

What is the most appropriate field term for a suspected fracture?

swollen, painful deformity

What assessment would you us for your patient that fell from a bed and suffered an injury to the left shoulder. His only complaint is pain in teh anterior shoulder. There was no loss of concious and the patient has an patent airway?

focused physical

What is an injury that causes teh bones at a joint to be seperated from each other?

dislocation

What is the most appropriate way to manage the wound if your patient has an evisceration from a gun shot wound?

cover organs with moist sterile dressing then occlusive dressing

What do you need to evaluate your patient with your patient was the restrained driver in a frontal impact at 65 MPH and as you examine, you note bruising over the abdomen which is tough and rigd?

hypovolemic shock

What is caused when you are palpating injured patients chest and note a crackling or bubbly sensation with your fingers?

air beneath skin

What i the appropriate care for a MVC patient that has bony crepitus and tenderness when you press on the pelvis and teh genitalia show swelling and discoloration?

bleeding from uretha

What 2 aspects cause the pulonary contusions to be life threatening injury?

1. reduces oxygen exchange 2. may cause pressure on heart

What is the current treatment for an emt to give with a flail chest?

stabilize chest with bulky dressing

What is your first action when your patient has an open wound to his left chest?

open airway and check for breathing

What is the most likely cause when your patient has suffered a closed injury and presents with decreased breath sounds on one side, JVD and hypotension?

Tension pneumothorax

What is a flail segment?

2 adjacent ribs fractured in 2 or more places

What organ is most likely to cause infection in a blunt injury to abdomen?

ruptured bowel

What is teh significance of this substances presence if your patient that has been struck in the eye by a piece of pipe and you see a clear gel-like substance on teh surface of the eye?

globe has been ruptured

What structure is not a part of the neck?

mandible

What is the seond action when your patient has recieved a knife wound to his lateral neck?

pressure dressing covered by an occlusive dressing taped on 4 sides

What is the best method for handling an avulsed tooth?

hold by the crown and transport in solution

What is the medical term for the white of the eye?

sclera

What is the most life-threatening problem associated with facial injuries?

airway obstruction

How do you treat your patient with splashed chemical into his left eye?

rinse eye copiously with water or saline: continue during transport

What is an injury that involves the eye being pulled from its socket?

extrusion

What is the best method for handling an object impaled in the eye?

stabilize with bulky dressing

What is your first action if your patient was assualted with a knife and recieved a laceration to the lateral neck and you note steady bleeding?

place a gloved hand over the laceration

What is the sign called when your patient was injured in a high speed motorcycle accident. His helmet was cracked and there is no evidence of trauma to his face but there is purple discoloration around both eyes?

racoon eyes

What has a patient suffered if he fell and struck his head on teh ground. He is unconcious for several minutes, then arouses. His nuero exam is normal.

concussion

What set of vital signs is most consistient with a severe head injury with increased intracranuial pressure?

160/104, p56,r34

What sign or symptom is most indicative of a severe head injury?

prolonged loss of concious

What is the most appropriate airway and breathing management if a patient was struck in teh face with a bat. He is unconcious with shallow respirations after applying c-spine precautions?

oropharyngeal with BVM

What is the most critical part of management for the patient with significant head trauma?

airway management

What functions are controlled by the brainstem?

breathing and heart rate

What is the appropriate rate of ventilations fo a patient with severe head injury?

10-12 / min

What is the response called when your patient suffered a severe head injury in a fall?

decerabrate posturing

What is teh term if the pressure inside teh cranium becomes severe and teh brain may be forced down through the foramen magnum?

herniation

Multilumen Airway- Removal etc

Doesn't require visualization for placement. There are 2 types, combitube and pharyngeotracheal lumen airway. Should not be used if the patient has a gag reflex, children under 14, adults shorter than 5', patients that have ingested a caustic substance, p

Laryngoscope Blade

it's purpose is to sweep the tongue out of the way and align the airway so the vocal cords can be visual and the ET tube passed through them. Blades are curved or strait and range in size 0-4. Blades contain a light to see better. Blade is detachable from

Signs IV site is infiltrated..

Edema or swelling at the catheter site, extremely slow IV flow, patient complaint of tightness and pain around the IV site.

Spike bag, prime line, and flush air.

IV bag.

Why are gastric tubes used pre-hospital?

used to relieve gastric distention caused by positive pressure ventilation prior to the advanced airway.

How can you tell the difference between sinus tach, and sinus rhythm on an EKG strip?

Can only tell be the number of waves on the strip. the waves should look the same but there are a lot more waves on the strip with sinus tachycardia.

If a patient is being adequately ventilated with bag-mask device attached to high-flow oxygen:

Endotracheal intubation may not be necessary.

Procedure for laryngeal mask airway

The LMA consists of two parts: the tube and the mask or cuff. After blind insertion, the device molds and seals itself around the laryngeal opening by inflation of the mask. The epiglottis contained within the mask or cuff. Comes in seven sizes and can be

Where does an IO needle go?

Proximal tibia

Vesicant agents are?

Blister agents. Such as mustard, lewisite, and phosgene oxide. Primary exposure is skin.

What is weaponization?

The creation of weapon from biologic agent generally found in nature and that causes disease. The agent is cultivated, synthesized, and or mutated to maximize the target population's exposure the germ.

The purpose of placards and labels on storage containers?

They are intended to give a general idea of the hazard inside a particular container or cargo tank. A placard identifies the broad hazard (flammable, poison, corrosive) to which the material inside belongs.

Responsibilities of medical branch of ICS?

Ensuring responder and public safety, achieving incident management goals, and ensuring the efficient use of resources.

a major benefit when using a multi-lumen airway device is that

maintenance of mask-to-face seal is not required

Approximately 10 seconds into an intubation attempt, you are unable to view the vocal cords. you should:

Direct your partner to apply posterior cricoid pressure.

If a patient if being adequately ventilated with bag-mask device attached to high-flow oxygen:

Endotracheal intubation may bot be necessary.

According to Department of Homeland Security Advisory system, the color orange indicates a ____ risk of terrorist attacks.

High

A weapon of mass destruction (wmd) is most accurately defined as:

Any agent used to bring about mass death, casualties, or massive infrastructural damage.

In the prehospital setting, gastric tubes are used primarily to:

Decompress the stomach

If an Iv line is found to be infiltrated, you should:

Remove the catheter and apply direct pressure.

What is the primary concern at the scene of a hazardous materials incident?

The safely of the EMT and Crew, patients and the public.

Whenever possible, a female sexual assault victim should be:

given the option of being treated by a female EMT.

A 26 -year- old female presents with heavy vaginal bleeding. She is conscious, but restless. Her blood pressure is 84/54 mm Hg, her pulse is 120 beats/min and weak, and her respirations are 22 breaths/min with adequate depth. She tells you that she insert

administer high-flow oxygen, place a sterile pad over her vagina, keep her warm, elevate her lower extremities, and transport without delay.

Potentially life-threatening consequences of PID include:

ovarian abscess and ectopic pregnancy.

Law enforcement personnel request your assistance to assess a 31-year-old female who was sexually assaulted. When you arrive at the scene, you find the patient sitting on a curb outside her apartment. She is conscious, alert, and crying. When you ask her

provide emotional support and visually assess her for obvious trauma.

Which of the following statements regarding rape is correct?

Rape is a legal diagnosis, not a medical diagnosis.

You are dispatched to a residence for a 40-year-old female who complains of lower abdominal pain, fever and chills, and a foul-smelling vaginal discharge. Which of the following additional assessment findings would increase your index of suspicion for PID

a shuffling gait when walking

During your assessment of a young female with nontraumatic vaginal bleeding, you note that her level of consciousness is decreased, her respirations are rapid and shallow, her skin is cool and moist, and her pulse is rapid and weak. You should:

assist her ventilations with a bag-mask device.

The third stage of labor begins when the:

the baby is expelled from the vagina

You have just delivered a full-term infant. His respirations are rapid and irregular, and he has a strong cry. What should you do next?

Assess the brachial or umbilical pulse.

Which of the following statements regarding the placenta is correct?

The placenta allows oxygen, carbon dioxide, and other products to transfer between the mother and fetus but does not allow blood to mix between the mother and fetus.

Eclampsia is MOST accurately defined as:

seizures that result from severe hypertension.

Which of the following statements regarding a breech presentation is MOST correct?

A breech presentation occurs when the buttocks are the presenting part.

A nuchal cord is defined as an umbilical cord that:

is wrapped around the baby's neck.

After the fetus has descended into the pelvis at the end of the third trimester, many mothers experience:

easier breathing

An infant is considered to be premature if it:

weighs less than 5 lb or is born before 36 weeks' gestation

The leading cause of maternal death during the first trimester of pregnancy is:

internal bleeding caused by a ruptured ectopic pregnancy

An abortion occurs when the fetus and placenta deliver before:

20 weeks

In contrast to a full-term infant, a premature infant:

has an even proportionately larger head.

During your visual inspection of a 19 -year- old woman in labor, you see the baby's head crowning at the vaginal opening. What should you do?

Apply gentle pressure to the baby's head as it delivers.

During delivery, it is MOST important to position your partner at the mother's head because:

the mother may become nauseated and vomit.

Which of the following occurs during true labor?

Uterine contractions become more regular

The term "bloody show" is defined as:

the small amount of pink-tinged mucus that is discharged from the vagina after expulsion of the mucous plug.

When the mother is experiencing a contraction, you should instruct her to:

take quick short breaths.

A pregnant trauma patient may lose a significant amount of blood before showing signs of shock because:

pregnant patients have an overall increase in blood volume

The vagina and the neck of the uterus comprise the:

birth canal

Braxton-Hicks contractions are characterized by:

alleviation of pain with movement or changing positions.

The ONLY indications for placing your gloved fingers in the vagina during delivery are:

breech presentation and prolapsed umbilical cord

The onset of labor begins with:

contractions of the uterus

By the 20th week of pregnancy, the uterus is typically at or above the level of the mother's:

belly button

Your 22 -year- old patient is in active labor. Upon visual inspection, you note that the infant's leg is protruding from the vagina. Appropriate management of this situation includes:

placing the mother supine with her head down and pelvis elevated.

Which of the following statements regarding gestational diabetes is correct?

In some cases, women with gestational diabetes require insulin injections.

After a baby is born, it is important to

ensure that it is thoroughly dried and warmed

Which of the following questions is of LEAST pertinence when determining whether a mother will deliver her baby within the next few minutes?

When are you due?

A newborn infant will usually begin breathing spontaneously within _______ seconds following birth.

15-30

Which of the following is a normal physiologic change that occurs in the mother's respiratory system during pregnancy?

increased respiratory rate and decreased respiratory reserve

If a newborn's heart rate is less than 60 beats/min following delivery, you should:

provide ventilations for 30 seconds.

The term primigravida refers to a woman who:

is pregnant for the first time

Placenta previa is MOST accurately defined as:

development of the placenta over the cervical opening

While examining a woman in labor, you see the umbilical cord protruding from the vagina. You should:

push the infant's head away from the cord

The amniotic fluid serves to:

insulate and protect the fetus

Preeclampsia MOST commonly occurs after the ____ week of gestation.

30th

When determining the frequency of contractions, you should time the contractions from the:

start of one to the start of the next

Following delivery of a full-term baby, you have properly cared for the baby and have clamped and cut the umbilical cord. During transport, you note that the mother is experiencing moderate vaginal bleeding. You should:

firmly massage the uterine fundus with a circular motion

If a baby is born at 7:52, the second Apgar score should be calculated at

7:57

The presence of thick meconium in the amniotic fluid indicates:

that the baby's airway may be obstructed.

Upon delivery of an infant's head, you note that the umbilical cord is wrapped around its neck. You should:

attempt to slip the cord gently over the infant's head.

You have just delivered a baby boy. His body is pink, but his hands and feet are blue. His heart rate is approximately 110 beats/min and his respirations are rapid and irregular. He has a weak cry when stimulated and resists attempts to straighten his leg

8

Which of the following is an indication of imminent birth?

crowning of the baby's head

Signs and symptoms of preeclampsia include:

headache and edema

From what internal female organ is the fetus expelled during delivery?

uterus

Common interventions used to stimulate spontaneous respirations in the newborn include all of the following, EXCEPT:

positive-pressure ventilations

Abruptio placenta occurs when

the placenta prematurely separates from the uterine wall.

Upon delivery of the baby's head, you note that its face is encased in the unruptured amniotic sac. You should

puncture the sac and suction the baby's mouth and nose

A 33-year-old woman who is 36 weeks pregnant is experiencing scant vaginal bleeding. During transport, you note that she suddenly becomes diaphoretic, tachycardic, and hypotensive. In addition to administering 100% oxygen, you should

place her in a left lateral recumbent position.

Vigorous suctioning of a newborn's airway is indicated if

there is meconium in the amniotic fluid

The umbilical cord

carries oxygen to the baby via the umbilical vein

Causes of infant death that may be mistaken for SIDS include all of the following, EXCEPT:

hyperglycemia

Febrile seizures in a child

may indicate a serious underlying illness

The components of the PAT are

appearance, work of breathing, and skin circulation

The MOST ominous sign of impending cardiopulmonary arrest in infants and children is

bradycardia

Which of the following findings is LEAST suggestive of child abuse?

consistency in the method of injury reported by the caregiver

When administering oxygen to a frightened child, it would be MOST appropriate to:

place oxygen tubing through a hole in a paper cup.

A 4-year-old female ingested an unknown quantity of liquid drain cleaner. Your assessment reveals that she is conscious and alert, is breathing adequately, and has skin burns around her mouth. You should

Monitor her airway and give her oxygen.

A common cause of shock in an infant is

dehydration from vomiting and diarrhea

Blood pressure is usually not assessed in children younger than _____ years.

3

Greenstick fractures occur in infants and children because

their bones bend more easily than an adult's.

The purpose of the pediatric assessment triangle (PAT) is to

allow you to rapidly and visually form a general impression of the child.

Common causes of seizures in children include all of the following, EXCEPT

hyperglycemia

You are dispatched to a residence for a child with respiratory distress. The patient, an 18-month-old female, is tachypneic, has sternal retractions, and is clinging to her mother. Her skin is pink and dry, and her heart rate is 120 beats/min. The MOST ap

administering blow-by oxygen and transporting the child with her mother

The MOST common cause of dehydration in pediatric patients is

vomiting and diarrhea

Early signs of respiratory distress in the child include

bradycardia

A 6-month-old male presents with 2 days of vomiting and diarrhea. He is conscious, but his level of activity is decreased. The infant's mother tells you that he has not had a soiled diaper in over 12 hours. The infant's heart rate is 140 beats/min and his

moderate dehydration

You respond to a skate park where a 10 -year- old male fell from his skateboard and struck his head on the ground; he was not wearing a helmet. He is responsive to painful stimuli only and has a large hematoma to the back of his head. After your partner s

assist his ventilations, be prepared to suction his mouth if he vomits, apply full spinal precautions, and prepare for immediate transport to a trauma center

A 6-year-old male presents with acute respiratory distress. His mother states that she saw him put a small toy into his mouth shortly before the episode began. The child is conscious, obviously frightened, and is coughing forcefully. You should

encourage him to cough, give oxygen as tolerated, and transport.

The EMT should be MOST concerned when a child presents with fever and

a rash

Cardiac arrest in the pediatric population is MOST commonly the result of:

respiratory or circulatory failure

After using the PAT to form your general impression of a sick or injured child, you should

perform a hands-on assessment of the ABCs.

You are dispatched to a local elementary school for an injured child. As you approach the child, you note that he is lying at the base of the monkey bars. He is unresponsive and there are no signs of breathing. You should:

stabilize his head and open his airway

Unless he or she is critically ill or injured, you should generally begin your assessment of a toddler

at the feet

Capillary refill time is MOST reliable as an indicator of end-organ perfusion in children younger than:

6 years

Blood loss in a child exceeding _____ of his or her total blood volume significantly increases the risk of shock.

25%

An 8-year-old female with a history of asthma continues to experience severe respiratory distress despite being given multiple doses of her prescribed albuterol by her mother. She is conscious, but clearly restless. Her heart rate is 130 beats/min and her

be prepared to assist her ventilations, transport at once, and request an ALS intercept en route to the hospital

Bruising to the _________ is LEAST suggestive of child abuse

shins

Death caused by shaken baby syndrome is usually the result of:

bleeding in the brain.

Which of the following is NOT a known risk factor of SIDS?

putting a baby to sleep on his or her back

Pale skin in a child indicates that the

blood vessels near the skin are constricted

The use of multiple prescription drugs by a single patient, causing the potential for negative effects such as overdosing or drug interaction, is called:

polypharmacy

You are assessing a 70-year-old female who complains of intense thirst, frequent urination, and dizziness. She has a history of type 2 diabetes, heart disease, rheumatoid arthritis, and gout. Her blood glucose level is 450 mg/dL. She is conscious, but con

hyperosmolar hyperglycemic nonketotic coma (HHNC).

General communication techniques with the elderly include

frequently asking the patient if he or she understands.

Because of the complexity of the older patient and the vagueness of his or her complaint, you should:

attempt to differentiate between chronic and acute problems.

You receive a call for a sick person. When you arrive, you find the patient, a 75-year-old male, lying unresponsive in his bed. His respirations are slow and irregular and his pulse is slow and weak. His daughter tells you that he fell the day before, but

a subdural hematoma

The purpose of the GEMS diamond is to:

help EMS personnel remember what is different about elderly patients.

Common causes of syncope in older patients include all of the following, EXCEPT:

vasoconstriction

Which of the following would be the MOST practical method of communicating with a hearing-impaired patient until his or her hearing aids can be located?

using a piece of paper and writing utensil to ask questions

You receive a call to a residence for an apneic 2-month-old male. When you arrive at the scene, the infant's mother tells you that her son was born prematurely and that his apnea monitor has alarmed 4 times in the past 30 minutes. Your assessment of the i

transport the infant to the hospital and bring the apnea monitor with you.

You are assessing a 440-lb man who complains of shortness of breath and lower back pain. The patient is conscious and alert, his blood pressure is 148/98 mm Hg, and his heart rate is 120 beats/min. Your MOST immediate action should be to

avoid placing him in a supine position if possible and administer oxygen.

Autism is MOST accurately defined as a

pervasive developmental disorder characterized by impairment of social interaction.

You respond to a residence for a 9 -year- old female with Down syndrome who is sick. When you arrive at the scene and assess the patient, you determine that she is unresponsive and has gurgling respirations. You should:

open her airway with a manual maneuver, suction her oropharynx, and insert a simple airway adjunct.

If suctioning of the tracheostomy tube is necessary, the EMT should:

attempt to use the patient's suction device first because it is probably already sized correctly.

911 system

-system for telephone access to report emergencies
-dispatcher takes info and alert EMS or fire or police
-"enhanced 911" has capability of automatically identify caller's number and location

abandonment

leaving a patient after care has been initiated and before patient has been transferred to someone with equal or greater medical training

ABC

airway, breathing, circulation

abdominal quadrants

4 divisions of the abdomen used to pinpoint the location of a pain or injury
RUQ,LUQ,RLQ,LLQ

acetabulum

pelvic socket into which the ball at the proximal end of femur fits to form the hip joint

acromioclavicular join

joint where the acromion and clavicle meet

acromion process

highest portion of the shoulder

acute myocardial infarction

heart attack

advance directive

DNR order

AED

automated external defribrilator

ALS

advanced life support

alveoli

microscopic sacs of lungs where gas exchange with bloodstream takes place

amniotic fluid

fluid-filled membrane where fetus develops

anatomical position

standard reference position for the body, body stands erect facing the observer with arms down at sides and palms of hands forward

anatomy

study of body structure

angina pectoris

chest pain caused by stress or exertion

anterior

front of body or body part

aorta

largest artery in body, transports blood from left ventricle to being systemic circulation

apnea

condition of not breathing

appendix

small tube located near the junction of the small and large intestines in right lower quadrant of abdomen

arteriole

smallest kind of artery

artery

any blood vessel carrying blood away from heart

artria

two upper chambers of heart

asthma

chronic obstructive lung disease triggered by allergy condition

asystole

absence of ventricular contractions

automaticity

ability of the heart to generate and conduct electrical impulses on its own

autonomic nervous system

division of peripheral nervous system that controls involuntary motor functions

AVPU

alert, responds to verbal stimulus, responds to pain stimulus, unresponsive

bilateral

on both sides

blood pressure

pressure caused by blood exerting force against the walls of blood vessels, usually arterial blood pressure is measured, diastolic/systolic

brachial artery

artery of upper arm, site of pulse checked during infant CPR

brachial pulse

pulse produced by compressing major artery of the upper arm

bradycardia

abnormal heart rate, usually 50 or fewer beats per minute

bronchi

two large sets of branches that come off trachea and enter lungs, right and left bronchi

BSI

body substance isolation

BVM

bag valve mask

C.V.A.

stroke

calcaneus

heel bone

capillary

thin-walled, microscopic blood vessel where the oxygen/carbon dioxide and nutrient/waste exchange with body' cells takes place

cardiac conduction system

system of specialized muscle tissues which conduct electrical impulses that stimulate the heart to beat

cardiac muscle

specialized involuntary muscle found only in the heart

cardiovascular system

system made up of heart( cardio) and blood vessels (vascular), circulatory system

carotid arteries

large neck arteries, one on each side of neck, that carry blood from the heart to head

carpals

wrist bones

central nervous system(CNS)

brain and spinal cord

central pulses

carotid and femoral pulses, which can be felt in central part of body

cervix

mouth of uterus

CISM critical incident stress management

comprehensive system that includes education and resources to both prevent stress and to deal with stress appropriately when it occurs

clavicle

collarbone

confidentiality

obligation not to reveal information obtained about a patient except to other health care professionals involved in patient's care

consent

permission from patient for care or other action by EMT

contamination

introduction of dangerous chemicals, disease, or infectious materials

COPD

chronic obstruction pulmonary disease

coronary arteries

blood vessels that supply the muscle of the heart(myocardium)

CPR

cardio pulmonary resuscitation

cranium

top, back, sides of skull

cricoid cartilage

ring-shaped structure that forms the lower portion of larynx

cyanosis

sign that adequate perfusion is not accomplished

DCAPBTLS

deformities, contusions, abrasion, punctures, burns, tenderness, lacerations, swelling

decontamination

removal or cleansing of dangerous chemicals and other dangerous or infectious materials

dermis

inner(second) layer of skin, rich in blood vessels and nerves, found beneath epidermis

designated agent

EMT or other person authorized by a medical director to give medication or give emergency care, extension of medical director's license to practice

diaphragm

muscular structure that divides chest cavity from abdominal cavity, major muscle of respiration

diastole

period of myocardial relaxation during which cardiac chambers fill

diastolic pressure

pressure in arteries when the left ventricle refilling

digestive system

system by which food travels through body and digested, or broken down into absorbable forms

distal

farther away from torso

DNR order

do not resuscitate order, legal document signed by patient and physician and states patient has terminal illness and does not wish to prolong life

dorsal

referring to back of body

dorsal pedis artery

artery supplying the foot, lateral to the large tendon of the big toe

duty to act

obligation to provide care to patient

dyspnea

difficulty in breathing

ectopic pregnancy

development of the fetus outside the uterus

emphysema

degeneration of alveolar structure

endocrine system

system of glands that produce chemicals called hormones that help to regulate many body activities and functions

epidermis

outer layer of skin

epiglottis

leaf-shaped structure that prevents food and foreign matter from entering trachea

epinephrine

hormone produced by body, as medication it dilates respiratory passages and is used to relieve severe allergic reactions

exhalation

passive process in which intercostal muscles(ribs) and diaphragm relax, causing chest cavity to decrease in size and air to flow out lungs

expressed consent

consent given by adults of legal age and mentally competent to make a rational decision in regard to their medical well-being

FBAO

foreign body airway obstruction

femoral artery

major artery supplying leg

femur

large bone of thigh

fetus

developing baby

fibula

lateral and smaller bone of lower leg

fowler's position

sitting position

gallbladder

sac on underside of liver that stores bile produced by the liver

good samaritan laws

series of laws varying in each state designed to provide limited legal protection for citizens and some health care personnel when they are administered care

hazardous-material incident

release of a harmful substance into the environment

HIPAA

health insurance portability and accountability act, federal law protecting the privacy of patient-specific health care information and providing the patient with control over how this information is used and distributed

humerus

bone of upper arm, between shoulder and elbow

hypertension

blood pressure above normal range

hypoglycemia

condition of low blood pressure

hypo-perfusion

inadequate perfusion of the cells and tissues of body caused by insufficient flow of blood through capillaries

hypotension

blood pressure below that considered normal

ilium

superior and widest portion of pelvis

implied consent

consent presumed by patient or patient's parent would give if they could such as for an unconscious patient or small child at school

inferior

away from the head

inhalation

active process in which intercostal muscles(ribs) and diaphragm contract, expanding the size of the chest cavity and causing air to flow into lungs

insulin

hormone produced by pancreas or taken as medication by many diabetics

insulin shock

condition of low blood pressure

involuntary muscle

muscle that responds automatically to brain signals but cannot be consciously controlled

ischium

lower, posterior portions of pelvis

joint

point where two bones come together

large intestine

muscular tube that removes water from waste products received from small intestine and removes anything not absorbed by the body toward excretion from the body

laryngectomee

patient who breathes through incision (partial or total removal of larynx) made in throat

larynx

voice box

lateral

to the side, away from midline

lateral malleolus

at lower end of fibula, seen on outer ankle

left atrium

receives oxygenated blood returning from the lungs

ligament

tissue that connects bone to bone

liver

largest organ, produces bile to assist in breakdown of fats and assists in the metabolism of various substances in body

LPM

liters per minute

lungs

organs where exchange of atmospheric oxygen and waste carbon dioxide take place

malleolus

protrusion on the side of ankle

mandible

lower jaw bone

manubrium

superior portion of sternum

maxillae

two fused bones forming the upper jaw

medial

toward midline

medial malleolus

lower end of tibia, seen on inner ankle

medical direction

oversight of patient-care of ems system

medical director

physician who assumes ultimate responsibility for the patient care aspects of EMS system

metacarpals

hand bones

metatarsals

foot bones

MI

myocardial infarction

mid-axillary line

line drawn vertically from middle of armpit to ankle

mid-clavicular line

line through center of each clavicle

midline

imaginary line drawn down the center of the body, dividing it into right and left

miscarriage

spontaneous abortion

multiple casualty incident MCI

emergency involving multiple patients

muscle

tissue that can contract to allow movement of a body part

musculoskeletal system

system of bones and skeletal muscles that support and protect the body and permit movement

nasopharynx

area directly posterior to nose

nervous system

system of brain, spinal cord, and nerves that govern sensation, movement, and thought

nitroglycerine

medication used for chest pain

NRB

non rebreather mask

obstructed airway

occurs when food bolus or other object prevents inspiratory/expiratory process

off-line medical direction

orders from medical director to EMT to give certain medications or do certain procedures without speaking to medical director or physician

on-line medical direction

orders from an on-duty physician given directly to EMT through radio or phone

OPA

oropharyngeal airway

OPQRST

onset, provocation, quality, radiation, severity, time

oropharynx

area directly posterior to mouth

pancreas

gland located behind stomach that produces insulin and juices that assist in digestion of food in duodenum of small intestine

pathogens

organisms that cause infection such as viruses and bacteria

perfusion

supply of oxygen to and removal of wastes from the cells and tissues of body as a result of the flow of blood through capillaries

peripheral pulses

radial, brachial, posterior tibial, dorsalis pedis pulses, which can be felt at peripheral (outlying) points of the body

pharynx

area directly posterior to mouth and nose, made up of oropharynx and nasopharynx

placenta abruption

premature separation of the placenta from the wall of the uterus

placenta previa

development of the fetus over the mouth of the uterus

plane

flat surface formed when slicing through a solid object

plasma

fluid portion of blood

platelets

components of the blood, membrane-enclosed fragments of specialized cells

PNS peripheral nervous system

nerves that enter and leave the spinal cord and travel between brain and organs without passing through spinal cord

posterior

back of body

posterior tibial artery

artery supplying the foot behind the medial ankle

PPE

personal protective equipment that protects the EMS worker from infection or exposure to danger of rescue operations

primary function of autonomic nervous system

regulate unconscious or involuntary body functions

prone

laying face down

protocols

-lists of steps ( assessments and interventions) to be taken in different situations
-developed by medical director of EMS system

proximal

closer to torse

PSI

pounds per square inch

pulmonary arteries

vessels that carry blood from the right ventricle of heart to lungs

pulmonary edema

fluid in lungs

pulmonary veins

vessels that carry oxygenated blood from lungs to left atrium of heart

pulse

rhythmic beats caused as waves of blood move through and expand arteries

quality improvement

process of continuous self review with the purpose of identifying and correcting aspects of the system that require improvement

radial artery

artery of lower arm

recovery position

lying on side, also called "lateral recumbent position

red blood cells

components of blood, carry oxygen to and carbon dioxide away from cells

respiratory system

system of nose, mouth, throat, lungs, and muscles that brings oxygen into body and expels carbon dioxide

right atrium

receives unoxygenated blood returning from the body

SAED

semi automatic external defribrillator

SAMPLE

signs & symptoms, allergies, medical history, past medications, last oral intake, event that led to injury/illness

scope of practice

set of regulations and ethical considerations that define the scope, or extent and limits of the EMT's job

small intestine

muscular tube between stomach and large intestine, divided into duodenum, jejunum, and ileum which receives partially digested food from stomach and continues digestion, nutrients are absorbed by body through its walls

sphygmomanometer

instrument used to determine arterial pressure with non-invasive technique

standard precautions

strict form of infection control that is based on assumption that all blood and other body fluids are infectious

standing orders

-policy or protocol issued by medical director that authorizes EMT's and others to perform particular skills in certain situations

sternum

breastbone

stomach

muscular sac between esophagus and the small intestine where digestion begins

stress

state of physical and or psychological arousal to a stimulus

subcutaneous layers

layers of fat and soft tissues found below the dermis

superior

toward the head

supine

laying on back

systole

part of cardiac cycle during which myocardial fibers contract and cardiac chambers empty

systolic blood pressure

pressure created in arteries when left ventricle contracts and forces blood out into circulation

T.I.A.

temporary signs of a stroke

tachycardia

abnormal heart rate, usually 120+ beats per minute

tarsals

ankle bones

tendon

tissue that connects muscle to bone

thorax

chest

tibia

the medial and larger bone of lower leg

torso

trunk of body, body without the head and extremities

trachea

windpipe", structure that connects the pharynx to lungs

trendelenburg position

position which the patient's feet and legs are higher than head

ulna

medial bone of forearm

uterus

womb

valve

structure that opens and closes to permit the flow of a fluid in only one direction

venae cavae

superior vena cava and inferior vena cava, these two major veins return blood from body to right atrium

ventral

referring to front of body

ventricles

two lower chambers of heart, right ventricle sends oxygen -poor blood to lungs and left ventricle sends oxygen-rich blood to body

venule

smallest kind of vein

vertebrae

33 bones of spinal column

voluntary muscle

muscle that can be consciously controlled

white blood cells

components of blood, produce substances that help body fight infection

xiphoid process

inferior portion of sternum

zygomatic arches

form the structure of cheeks

Abortion

The delivery of the fetus and placenta before 20 weeks; miscarriage

wrapped in a sterile dressing, placed in a plastic bag, and kept cool

Your unit has arrived on the scene of a patient who stuck his hand in a snowblower and accidentally amputated his right middle and ring fingers. When caring for a patient with an amputation, the amputated part should be:

Your unit has arrived on the scene of a patient who threw gasoline on his grill in an attempt to light the fire faster. There was an explosion, and he has thermal burns on most of the front of his body, causing blisters and discomfort. Once you are sure t

partial thickness

Your unit is on the scene of a warehouse fire. A firefighter just carried out a patient who has burns completely around both of his arms and on the front of his chest. His vitals are respirations of 26, pulse of 110 and BP of 110/70. Circumferential burns

constrict the skin, interrupting circulation to the distal tissues

Bones are covered with a strong, white, fibrous material called the

periosteum

Your unit is assessing an 8-year-old child who fell from the monkey bars at a local park. The patient has vital signs of respirations 24, pulse 100, and BP of 96/P. He did not strike his head or lose consciousness, but his lower right leg may be broken. S

can prevent additional blood loss, pain, and complications from nerve and blood vessel injury.

Your unit is on the scene of a car vs. motorcycle collision. After assuring the scene is safe and conducting an initial assessment, you determine from your rapid trauma exam that the patient has road rash and swollen deformed right thigh. Since his vital

a femur fracture

You are on the scene with a patient who sustained a serious head injury from striking the side of his head. His vital signs are respirations of 24, pulse of 56 and BP of 170/90. You use the Glasgow Coma Scale as a method of

ongoing neurological assessment

Which of the following is not a common cause of hypoperfusion, or shock, in infants and children?

cardiac compromise

The viral disease that results in inflammation of the larynx, trachea, and bronchi in children is called:

croup

Unless there is a specific exception for your state, the medications carried on the ambulance should include

oral glucose

Your unit has just arrived on the scene of a high-speed collision. You determine that it will be necessary to detour the traffic away from the scene. When placing flares at a motor vehicle collision site, things to remember include:

avoiding spilled fuel and dry vegetation

Your unit is on the scene of a multiple vehicle collision. Your patient was found on the ground, lying besides his car and a bystander states he witnessed the patient exit the car, and take two or three steps then fall to the ground. After assessing the p

elevate the legs for spinal shock

An example of a "shockable" rhythm, that is, one for which defibrillation is effective, is

rapid ventricular tachycardia

Pulseless electrical activity is a condition in which:

there is only electrical activity of the heart, without pumping action

Your unit is on the scene of a 23-year-old female office worker who co-workers state was acting like she was intoxicated for the past hour. Apparently, from her Medic Alert� bracelet, she has a history of diabetes. Typical signs and symptoms of hypoglycem

altered mental status, leading to possible unconsciousness

Anything that causes an allergic reaction is called a(n):

allergen

Activated charcoal is given to patients who have ingested a poison or overdose because it:

adsorbs or binds to many poisons

Drugs capable of producing stupor or sleep, which are also used to relieve pain and to quiet coughing are

narcotics

The body loses heat in several ways. One way is when currents of air pass over the body, carrying heat away. This is called

convection

If a heat emergency patient has hot skin, whether moist or dry, the

patient may have heat stroke, which is considered a true emergency

Your unit has arrived on the scene of a patient who attempted to take his life by slashing his wrist. He is a 20-year-old, ambulatory at the scene and pacing back and forth with two seriously bleeding wrists. When dealing with a potential or attempted sui

his/her own safety

The developing baby, or fetus, grows inside the mother's

uterus

The developing fetus floats in a thin membranous bag filled with

amniotic fluid

Once the baby's head delivers, if you see the umbilical cord around the baby's neck, you should:

place two fingers under the cord and remove it from around the baby's neck

Which of the following is true about veins?

Veins have one-way valves that prevent blood from flowing in the wrong direction

Hypoperfusion, or shock, can be caused by blood loss. Severe blood loss results in

inadequate fluid to fill the capillaries and inadequate oxygen and nutrients to the cells

When transmitting on a radio, you should

listen to the channel before starting to talk, hold the microphone 2 to 3 inches from your lips as you speak, depress the microphone and wait a second before talking

The standardized medical abbreviation for loss of consciousness is

LOC

An EMT-Basic may assist with which injectable liquid?

Epinephrine

The method for reporting breathing rate is in respirations per:

minute

On a child over eight years of age, which is the primary site for evaluating a peripheral pulse?

radial

Cool, moist skin may indicate anxiety or

shock

Drug use can cause the pupils to become

either dilated or constricted

You are treating a patient complaining of chest pressure. You completed your assessment, applied oxygen, and are transporting him to the hospital, now that the pain has subsided. His vital signs are BP 134/70, HR 78, and RR22. How often should you reasses

Every 15 minutes

A turbulent breathing sound that may be present during inspiration is called

crowing

The parts of the lower airway are the

trachea, bronchi, bronchioles, and alveoli

Your unit is on the scene of a 22-year-old male patient who has taken a large number of pills in an attempt to kill himself. If a patient has stopped breathing, or has inadequate breathing, the EMT-Basic should provide:

artificial ventilation

Abandonment is acceptable only if the EMT-Basic provider is

in fear of his or her own safety

Expressed or informed consent cannot be given by a patient who is:

mentally incompetent

An insufficiency in the supply of oxygen to the body's tissues is called:

hypoxia

In the event of copious thick secretions or vomiting, you may consider

removing the suction tip and using the large-bore tubing to suction the patient.

The ambulance should be inspected every

tour or shift

Unconscious patients with no suspected spinal injury should be placed in the _____ position.

recovery

An extremity lift or the draw sheet method of moving a patient is appropriate only for a patient:

with no suspected spine injury.

The proper use of your body to facilitate lifting and moving is called body

mechanics

In the acronym "SAMPLE" each letter stands for an element of the patient's history. For example, the "A" stands for

allergies

The pressure created when the heart contracts forcing blood into the arteries is called___blood pressure. When the heart relaxes, the pressure remaining is called ____blood pressure.

systolic:diastolic

When your patient inhales, the intercostal muscles:

and diaphragm contract

The division of the nervous system that controls involuntary motor functions and affects digestion and heart rate is the ______ nervous system.

autonomic

The body's largest artery is the

aorta

Roles and responsibilities of the EMT-Basic include

patient assessment, lifting and moving patients, transport of patients.

Your unit has been dispatched to a local bar in which there has been a fight. Your first priority as an EMT-Basic when called to a scene involving violence is

to be certain the scene is safe before entering

Personal protective equipment (PPE) for body substance isolation (BSI) may include

latex or vinyl gloves

You are caring for a terminally ill patient who tells you, "It's okay. I'm not afraid." This is called the ______stage.

acceptance

The front half of the body is called:

anterior

A list of steps, such as assessment and interventions, to be taken by an EMT-Basic in specific situations, is called a(n)

protocol

In order for a patient to refuse care or transport, the conditions that must be met include

the patient must be fully informed of the risks.

A thrombus, which has broken loose, moving with blood flow is called a(n)

embolism

Your unit is on the scene of a 67-year-old male who has been complaining of substernal chest pain for the past two hours. A contraindication to assisting the patient in taking nitroglycerin tablets is the:

systolic blood pressure is less than 100 mmHg

A common symptom in patients with cardiac compromise is an abnormal heart beat. A heart rate that is too slow is called

bradycardia

The most important drug the EMT-Basic can give to a patient with a heart problem is

oxygen

You are on the scene of a 21-year-old female patient who is wheezing and in obvious distress. Her co-worker states that she has a history of asthma and carries a bronchodilater. An asthma attack is often triggered by

an allergic reaction, emotional stress, strenuous exercise

Signs of moderate to severe respirtory distress include all of the following, except

increased pulse rate in children

The face is comprised of 14 irregular shaped bones that are all fused into immovable joints, except

mandible

Indirect medical direction involves the following of written standing orders called:

protocols

Which of the following is true regarding the airway structure of an infant or child?

A child's tongue is proportionately larger than an adult's

When assessing breathing for quality, normally we should observe

the use of accessory muscles to breathe

The structure that divides the chest cavity from the abdominal cavity is the

diaphragm

Epinephrine is administered to reverse the symptoms of severe allergic reaction. It also increases the heart rate. This action of the drug is known as a

side effect

Bronchodilators an EMT-Basic can assist a patient in taking come in the form of a(n)

inhaler

The medication that absorbs some poisons and is used to treat poisonings and overdoses that are swallowed is

activated charcoal

Writing in the prehospital care report that you provided an intervention that you actually did not provide is called

falsification

When noting times on your prehospital care report you should use the times

as given by the dispatcher

Guidelines for interpersonal communication include

being aware of your position and body language.

When asking about your patient's history of present illness, the letter "P" in the memory aid OPQRST means:

provokes

All of the following are considered a significant mechanism of injury, except

fall off a 6-foot ladder

Your unit is on the scene of a 56-year-old male patient who was thrown from a vehicle during a high-speed crash. When conducting your initial assessment, due to the significant mechanism of injury, instead of a focused physical exam, you should perform a(

rapid trauma assessment

If the mechanism of injury exerts great force on the upper body, or if there is any soft tissure damage to the head, face, or neck, you should

apply a cervical collar

The "L" in DCAP-BTLS stands for:

laceration

If your patient is dark-skinned, check the skin color of the:

lips

Part of assessing the ABCs is assessment of the patient's circulation. Check the pulse of a responsive adult patient at the ____ artery.

radial

The AVPU levels of responsiveness include:

painful

The initial assessment has six parts. These include

determining transport priority

The initial assessment is designed to identify immediately life-threatening conditions. These conditions include

severe bleeding

Injury caused by a blow that strikes the body but does not penetrate the skin or other body tissues is called _____ trauma.

blunt force

The "up and over" pattern is an injury type common in ______ collisions.

head-on

Your unit is on the scene of a 59-year-old male patient who was assaulted while attempting to use the ATM outside the local bank. It appears that the patient's neck was slashed. External bleeding from the large veins in the neck can be dangerous because t

venous pressure may be lower than atmospheric pressure, and air may be sucked into the vein

Air in the chest cavity between the lung and chest wall is called a(n)

pneumothorax

Your unit is on the scene of a patient who was assaulted with a knife. He was slashed from side to side with a carving knife. When abdominal organs protrude through a laceration, this is called a(n)

evisceration

Adding a reservoir to a bag mask device increases the oxygen delivered to your patient from 50 percent to nearly _______ percent.

100

You are on the scene of an unconscious 33-year-old male patient. You have no reason to believe that there was any trauma involved in the call. As you insert an oropharyngeal airway into your patient, he begins to gag. You should

remove the airway

The diameter of the patient's little finger is a common method of sizing a nasopharyngeal airway (NPA). Another method of sizing is to measure

the nostril to the earlobe

When a D cylinder of oxygen is full its pressure should be _______ psi.

2,000 to 2,200

Most airway problems are caused by

the tongue

When suctioning an infant or child, use a rigid tip but be careful not to

touch the back of the airway

When performing artificial ventilation on an infant you

must be careful to avoid excessive pressure or volume

During the initial assessment of a trauma patient, which of the following would not be criteria for a rapid transport

An allergic reaction

The abdomen should be inspected for any abnormal ____ or discoloration.

distention

Any patient who presents with an altered mental status may be suffering from ______, which can be assessed with a drop of the patient's blood and a(n):

a low blood sugar level/glucometer

Which of the following is an example of pertinent past medical history for a patient presenting with chest pain?

Cardiac bypass five years ago

Assessment, intervention, and ______ of the patient, is the key to all emergency care provided in the prehospital environment.

reassessment

The EMT-Basic may only administer or assist with administration of medication that is:

identified in local protocols

Every medication that the EMT-Basic will administer, or assist the patient with, requires:

an order from medical direction

The medication most commonly administered by the EMT-Basic is

oxygen

Medications administered by injection, such as an epinephrine auto-injector, should be injected:

only in specific muscle groups

When obtaining information about the onset of dyspnea in a patient, the EMT-Basic would ask any of the following questions, except:

On a scale of 1 to 10, how bad is your breathing difficulty?

A _______ is a device that consists of a canister of aerosolized medication and a plastic container that is used to deliver a medication by inhalation.

metered dose inhaler

A child in respiratory distress will have more prominent retractions than an adult would in the same situation because:

the child's chest wall muscles are not well-developed

Of the following conditions, which one is the most frequently seen in patients with cardiac dysfunction leading to congestive heart failure?

pulmonary edema

All of the following are late signs of pulmonary embolism, except

increased blood pressure

A 40-year-old male is complaining of chest discomfort, shortness of breath, and weakness. The EMT-Basic should immediately suspect

acute coronary syndrome

When caring for adolescents, it is important to remember that adolescents

want to be treated like adults and assessed away from parents

The total blood volume of an average preschooler is approximately ____ percent of the total body weight.

8

Your 2-year-old female patient is in obvious respiratory distress with stridor and audible wheezing. The most likely cause might be

upper airway obstruction

The signs and symptoms of shock (hypoperfusion) in children include

pale, cool, clammy skin

Which of the following is the most common cause of seizures in infants?

high fever

A child who has been struck by a vehicle may present with a triad of injuries, including the legs

head and abdomen

Indications that your 8-year-old male trauma patient may be abused would include

evidence of past or repeated burns

The ______ branch off from the base of the aorta and are responsible for supplying the heart itself with a rich supply of oxygenated blood.

coronary arteries

The major difference between the fully automated AED and the semi-automated AED is that the

the fully automated AED will deliver the shock itself.

The EMT-Basic is applying AED pads to a patient and discovers that the patient is wearing a transdermal patch on the right upper chest. The EMT-Basic should

remove the patch, with gloved hands, and wipe the site. Then apply the patch.

A type of medication used to reduce the amount of fluid in the body due to chronic heart failure is a:

diuretic

A condition where there is a build-up of fluid in the body from the pump failure of the heart is called:

congestive heart failure

The heart contains specialized _____, known as the cardiac conduction system, which allows it to generate electrical impulses.

conductive tissue

Which of the following patients might the EMT-Basic expect to find with an altered mental status?

A 28-year-old male with a history of seizures, who just stopped seizing.

For the patient experiencing an allergic reaction, the EMT-Basic should focus the physical exam on

complaints involving the airway, breathing, and circulation.

Epinephrine is the drug of choice for severe allergic reactions because:

it constricts blood vessels to improve blood pressure, it relaxes smooth muscle in the lungs to improve breathing, it works to reverse swelling and hives

Which of the following statements about poison ivy is true?

You do not need direct contact with the plant to have a reaction.

Your unit is on the scene with a group of skiers who were just found after being lost for the past 12 hours. Upon assessment you find that one of the skiers has frostbite in the toes on his left foot. Care for this patient will include:

splinting the left foot

Initially unconscious after being involved in a motor vehicle collision, the patient awakens to find that he cannot feel or move either of his legs. The patient most likely has

sustained damage to the spinal cord.

If a first responder is holding manual in-line spinal stabilization of a patient with a suspected spinal injury upon arrival, the EMT-Basic should:

instruct the first responder to continue stabilization and not let go.

Your unit has responded to the scene of a two-car collision. One patient is seated behind the wheel and the crew is preparing a KED. Another patient with a suspected spinal injury is found standing. You should:

immobilize the patient while standing and/or use the standing takedown technique.

During a rapid trauma exam, the EMT-Basic discovers leaking cerebrospinal fluid (CSF). Which of the following locations on the body might this be found?

any orifice in the head

Early signs that a person may have lost control and may become violent include all of the following, except

withdrawal

Traumatic factors that could mimic a behavioral emergency include

significant blood loss from internal bleeding

The structure of the umbilical cord and the blood traveling through it prevent the cord from:

kinking

Your unit has responded to a call for a 19-year-old woman in labor to assist with the delivery of her full-term infant. Now that both mother and child are stable, it is time to manage the umbilical cord. You should

place two clamps on the cord and cut between them

Your unit was dispatched to the shopping mall where you find a 30-year-old patient who is having a seizure. She is obviously pregnant. When a pregnant woman experiences a seizure:

is can be a life-threatening emergency for the mother and the fetus

Allowing the infant to breast-feed soon after childbirth

stimulates the release of oxytocin, which causes uterine contractions

You are on the scene of an 18-year-old female who you found in the bathroom with an infant partially delivered. You quickly assist in the delivery of a baby boy. One of the most important things the EMT-Basic can do in the care of the newborn is:

keep the infant warm and dry

After the amniotic sac ruptures, the umbilical cord, rather than the head may be the first part presenting at the vaginal opening. This is called a

prolapsed cord

During the delivery of multiple fetuses, if the second infant has not delivered within 10 minutes, the EMT-Basic should

transport the mother to the hospital for delivery of the second infant

vehicle is considered stable when

it can no longer move, rock or bounce

Using a police or other emergency vehicle escort enroute to the hospital is

dangerous and should be avoided

Which of the following is not considered a warning device approved for ambulances?

Public address (PA) system

As an early priority at the scene of any hazardous materials emergency:

safety zones are established for rescue operations.

At the scene of a MCI, sectors are established to

provide an orderly means of decision making

Trending is a step in the __________________ assessment.

ongoing

Your unit has responded to the scene of a patient who is in respiratory distress. He is a 22-year-old and you have obtained vital signs of respirations 34, pulse 110, and BP of 160/88. Signs of insufficient supply of oxygen to the body's tissues in an adu

pink mucous membranes.

When suctioning a patient's airway, you should suction slowly as you

withdraw the tip from the patient's mouth.

The first element of the scene size-up is scene safety. The EMT-Basic's first safety concern is for that of:

one's self

Your unit is on the scene of a call for breathing difficulty, which occurred in an adult care facility. The patient is a 69-year-old woman in respiratory distress. Her vitals are stable but she has a persistent productive cough. The disease in which the b

chronic bronchitis

Upon arrival at the scene of a motor vehicle collision you assure that the scene is safe. While conducting the initial assessment, once your patient's life-threats are under control, you will next

decide on the patient's transport priority

The assessment steps you take for the purpose of discovering and dealing with life-threatening problems are called the ______ assessment

initial

Finding out what is or may be wrong with a medical patient is similar to identifying the mechanism of injury with a trauma patient. This is called

nature of illness

What caused an injury (e.g., rapid deceleration causing the knees to strike the dashboard) is known as the

mechanism of injury

When using the head-tilt, chin-lift maneuver to open an unconscious patient's airway, your hand(s) should be placed on the

forehead, with the fingertips of the other hand under the jaw

When two rescuers are providing bag mask ventilation

one squeezes the bag while the other maintains a mask seal

The diaphragm is a

muscular structure that divides the chest cavity from the abdominal cavity

The non-breathing adult patient should be ventilated by the EMT-Basic at a rate of once every _____ seconds

5

All of the following conditions may require oxygen administration, except:

a twisted ankle injury

Suction is applied after the rigid tip is in place and

while withdrawing the catheter or Yankauer.

When ventilating a patient with a stoma, the EMT-Basic should:

ventilate by sealing a pediatric-sized mask over the stoma

All of the following are hazards of oxygen cylinder handling, except

oxygen regulators should be tightened with a steel wrench.

Before connecting the regulator to an oxygen tank the EMT-Basic should:

crack" the cylinder valve slightly to clear any dirt from the delivery port

In-line stabilization is performed under which circumstances?

suspected spinal injury

Which of the following is the mnemonic for assessment of a patient's mental status?

AVPU

An initial assessment (IA) of the circulation includes all of the following, except

obtaining a blood pressure

Which of the following is not a sign of inadequate breathing?

chest wall symmetry

A major factor to consider when assessing the condition of the skin would be the

effects of the environment on the patient

Apparent in some patients with chronic respiratory diseases, ______ is designed to keep the airway pressure in the smaller bronchioles higher during exhalation so they do not collapse

pursed lip breathing

A childhood respiratory condition that presents with drooling, while the child sits straight upright, jutting his neck out, is called

epiglottitis

While assessing a 76-year-old male who is lying in bed, the EMT-Basic hears gurgling coming from the patient's throat, and the lungs sound like fluid. The patient is cyanotic and gasping for breaths. What treatment should be done immediately?

Sit the patient upright and begin positive pressure ventilations

A significant sign of chest trauma, in which an area of the chest moves inwardly during inhalations and outward during exhalation, is called

paradoxical motion

When using a flow-restricted, oxygen-powered ventilation device, the oxygen concentration being delivered is ____ percent.

100

Before the EMT-Basic can assist a patient with an MDI he/she must

administer oxygen, verify the prescription is the patient's, get approval from medical control.

A severe asthmatic attack that does not respond to either oxygen or medication is called

status asthmaticus

Prolonged ____ can lead to too much carbon dioxide elimination, resulting in muscle cramps in the hands and feet.

hyperventilation

Chest pain caused by a narrowed coronary artery, which cannot supply enough oxygen to a portion of the heart during exertion is called

angina pectoris

The medication nitroglycerin is often prescribed to patients with a history of chest pain. The action of nitroglycerin is:

dilation of blood vessels

Chaotic electrical activity originating from many sites in the heart is called

V-Fib

Which of the following is true regarding asystole?

The heart has ceased generating electrical impulses

A 44-year-old female patient with chronic hypertension, a mild cough, and allergies was working in the yard when she suddenly felt a severe substernal chest pain radiating to her neck and down her left arm. Her husband called EMS. What is her chief compla

Pain radiating up the neck and down one arm

Your unit has responded to the local ski resort's first aid room. A ski patroller tells you that Sally has an open fracture of the right femur after colliding with a tree while skiing. The wound is bleeding and painful. How often should you reassess her v

every 5 minutes

Your unit is on the scene of a 34-year-old female who is in respiratory distress. During your assessment, you note she is using her accessory muscles to breathe. Examples of accessory muscles of breathing are the______muscles.

sternocleidomastoid

Serious blood pressure problems are generally considered to exist when the

systolic pressure falls below 90 mm/Hg.

You are on the scene of a 38-year-old male who fell off a ladder, striking the right side of his head. His vital signs are currently stable and you note equal strength in all four extremities. When checking a patient's pupils, shining a bright light into

constrict

Skin color and temperature can provide clues to significant blood loss in a patient because:

constriction of the blood vessels causes the skin to become pale

The patient experiencing inadequate breathing may show which of the following signs?

Movements associated with breathing limited to the abdomen

The rate of respirations is classified as normal, rapid, or slow. The normal range for an adult at rest is between ___breaths per minute.

12 and 20

When measuring the radial pulse, place:

your first three fingers on the thumbside of the patient's wrist just above the crease.

The term automaticity refers to the property of the heart that allows it to

generate and conduct electrical impulses on its own.

The blood vessel through which oxygen-rich blood is returned to the heart from the lungs is the:

pulmonary vein

You are treating a 72-year-old male patient who has the signs and symptoms of a heart attack. His vital signs are respirations of 24, pulse of 140, and BP of 98/64. A complication of a heart attack is mechanical pump failure. This can lead to

cardiac arrest, shock, fluid in the lungs

Hardening of the arteries," a stiffening of the artery wall resulting from calcium deposits, is called:

arteriosclerosis

One reason patients with cardiac compromise think they might have the "flu" is because the symptoms include

sweating with nausea

An example of trending a patient's condition would be

taking multiple sets of vital signs and comparing

The first step in the ongoing assessment of a critical patient is to:

repeat the initial assessment

All of the following are signs and symptoms that indicate the AED should be applied to a patient, except

unable to speak

After delivering one shock with the AED to a patient found not breathing and pulseless, the AED states, "no shock advised." The EMT-Basic should

continue with the steps of CPR

Which of the following types of patients are known to present with atypical presentations of acute coronary syndromes?

the elderly,diabetes, women

The EMT-Basic has assisted a patient with chest pain to take two nitroglycerin tablets, which have been prescribed to the patient. The patient's chest pain is less, but still present, and the last set of vital signs are: respiratory rate of 18, pulse of 8

stop giving nitroglycerin and check with medical control for directions.

In a normal heart the electrical impulse is generated in the _____ at the sinoatrial node.

right artium

The severity of shock is classified into three catagories. When the body is able to maintain blood pressure and perfusion, it is called _______ shock.

compensated

A crush injury occurs when force is transmitted from the body's exterior to its internal structures. An example of a crush injury is a

ruptured spleen

Your unit has arrived on the scene of a slashing. The police are on the scene advising; the scene is secure, and the patient is bleeding severely from a neck wound. During your initial assessment, one of your first steps is to control the bleeding from th

occlusive dressing

Your unit responds to a basketball court where you find a 22-year-old male who went up for a shot and came down hard-breaking his right thigh. His vital signs are respirations of 22, pulse of 100, and BP of 120/76. A traction splint is applied to a femur

reduce muscle spasms

Bones are living tissue and bleed when they are injured. A simple closed tibia-fibula fracture can result in blood loss of

500 cc

Your unit is on the scene of a local nursing home where EMS was called for an elderly patient who fell. Upon your assessment it appears as if the patient may have a painful right hip. Stabilization of a hip fracture is accomplished by

binding the legs together with a blanket placed between them.

Some parts of the spine are more susceptible to injury than others. The two parts most susceptible are the

cervical and lumbar vertebrae

A patient is complaining of low back pain from an old herniated disk injury. You find him standing in the kitchen holding onto the countertop, and he is in a lot of pain. What would be the best method to immobilize him?

Use the standing takedown onto a long backboard.

Your unit is on the scene at the local YMCA basketball courts. You are assessing a 14-year-old male who was slammed to the floor, with no loss of consciousness, while playing basketball. The patient is groggy and has a headache. Which of the following inj

concussion

You are using the Glasgow Coma Scale (GSC) to monitor the mental status of a head-injured patient. You assess the patient for verbal response, motor response, and

eye opening

Systolic blood pressure is a measurement of pressure in the

arteries while the heart is contracting

The nerves that carry messages from the brain to the body are the _______ nerves.

motor

You are assessing an unconscious 29-year-old male construction worker who fell 20 feet to the ground from a platform. You find blood and clear fluid draining from his nose. His vitals are: respirations of 24, pulse 64 and BP 160/92. You should suspect

a skull fracture

You unit is on the scene of a 59-year-old male patient who is complaining of chest pain. You have completed an initial assessment and are conducting the focused history and physical exam. One way to remember the questions asked to obtain a history of pres

OPQRST

Your unit is on the scene of a 22-year-old female who was thrown from her motorcycle. You have completed the initial assessment and a rapid trauma exam. An easy way to remember what you are trying to find when you assess a suspected injured area of the pa

DCAP-BTLS

Bullets cause damage in two ways. One is damage caused directly by the projectile. The other is called

cavitation

You have arrived on the scene of a patient who was shot with a handgun in the right upper leg. As you begin your initial assessment, it is clear that the patient has serious external bleeding, which needs to be handled right away. If, after you have appli

apply additional dressings on top of it and hold firmly in place

The layer of the skin that consists of mostly fatty cells and connective tissue is called the:

subcutaneous layer

You have just arrived on the scene of a 38-year-old male who was accidentally shot in the right upper chest when his rifle went off as he was climbing down from a tree stand. The hunter has an open chest wound and is in respiratory distress. How should op

Chest wounds should be covered immediately with a gloved hand

A third-degree burn is also called a_____burn

full-thickness

A patient has sustained second-degree burns on the front of his torso and the front of his right leg. What percentage of his body surface is burned?

27

The rapid trauma exam is performed:

when a trauma patient is unconscious.

The _____ takes more time to complete than other exams and is usually performed when all ongoing critical care has been managed.

detailed physical exam

After being involved in a moderate speed motor vehicle collision, the driver of the vehicle is presenting with temporary concussion-like insult to the spinal cord that causes effects below the level of the injury. The EMT-Basic should suspect

spinal shock

Sizing of an immobilization collar to a patient is based on

the design of the device

Signs and symptoms of a diabetic emergency include all of the following, except

decreased heart rate

The distinguishing factor between anaphylaxis and a mild allergic reaction is that, in anaphylaxis, the patient will have

respiratory distress

The use of activated charcoal is contraindicated for the patient who has ingested

an oven or drain cleaner

One way the body cools itself is by producing perspiration. Perspiration on the skin vaporizes. This method of cooling is called

evaporation

heat emergency patient with moist, pale, normal-to-cool skin may have which of the following signs and symptoms?

Muscular cramps, usually in the legs and abdomen

The general term for poisons produced by living organisms, such as animals or plants, is

toxin

Your patient's abnormal behavior may be due to a medical or traumatic condition. These problems include

inadequate blood supply to the brain, lack of oxygen, low blood sugar

Risk factors associated with suicide include all of the following, except

ages 25 to 35 years which show a high suicide rate.

When dealing with a violent or potentially violent patient, you should

not let the patient stand between you and an exit

What type of emergencies may require the EMT-Basic to administer activated charcoal?

Overdose

Oral glucose is most commonly administered in what form?

gel

Because nitroglycerin causes a slight drop in blood pressure, low blood pressure is a(n)_________ to give the drug.

contraindication

Albuterol, Ventolin, and Proventil are names of prescribed medications your patient may have to treat:

breathing difficulty

Every drug is listed in the U.S. Pharmacopoeia by its generic name. An example of a generic name for a medication is

epinephrine

Upon arrival at the scene of a patient, you determine the scene is safe and you have enough help available. You begin to question the patient. What the patient tells you is wrong is the

chief complaint

The focused history and physical exam of the medical patient is guided by the information obtained from the _____ questions.

OPQRST and SAMPLE

An EMT-Basic administers medications

under the direct order of a licensed physician.

The route and form in which a drug is given is referred to as the

method of administration

Following the administration of a medication, it is necessary to ____ and note any changes in the patient's condition.

perform an ongoing assessment

A medication that has a past due expiration date

should not be used

You have just arrived on the scene of a construction site. Apparently one of the workers, a 26-year-old male, is having difficulty breathing, and expressed that he has food allergies. The EMT-Basic should suspect that a patient who has a known serious all

have a prescribed epinephrine auto-injector

Signs and symptoms associated with anaphylaxis/allergic reaction that involve the cardiovascular system may include all of the following, except

nausea and vomiting

You respond to the scene of a poisoning. According to the bystanders, the patient drank a very strong quantity of an acid about 30 minutes ago. If an acid is ingested, a majority of the chemical burn will occur in the

stomach

You have responded to an alleyway on a very cold night. Your patient is an elderly patient who wandered off from the adult home and has been laying collapsed in the snowbank for at least an hour. Care of the patient with generalized hypothermia includes e

rough handling may induce a cardiac dysrhythmia

To maintain an open airway in the pediatric patient, it is important to

position the child's head in a more neutral position than an adult's

Signs of epiglottitis include the patient

sitting in the tripod position, sitting very still, but working hard to breathe, complaining of painful swallowing

Prior to delivery, if the mother is in the supine position she may develop dizziness and a drop in her blood pressure. This is called

supine hypotension syndrome, caused by the weight of the baby compressing on the inferior vena cava.

Which of the following is true regarding the delivery of a baby.

Keep the baby at the level of the mother's vagina until the cord is clamped and cut.

Excessive pre-birth bleeding may be the result of placenta previa. This condition occurs when the

placenta is formed in an abnormal location that will not allow for a normal delivery of the fetus.

The pulse rate of an infant or child with severe respiratory difficulty will

decrease

If a newborn's heart rate is below 100 beats per minute, the EMT-Basic should

provide artificial respirations at a rate of 40 to 60 per minute.

You unit has arrived on the scene of a private residence where you are met by a 23-year-old woman who is nine months pregnant. She tells you this is her third child and that her water just broke. The "bag of waters" is also known as the

amniotic sac

Since newborns can lose heat quickly, an EMT-Basic should

dry and wrap the infant in a warm blanket.

When caring for a 3-year-old patient it is important to remember that

fears needles and instruments

Which of the following statements is true regarding pediatric patients in respiratory distress?

nasal flaring and audible wheezing are early signs.

An important concept to understand about shock (hypoperfusion) in infants and children is that

their bodies compensate for blood loss for a long time.

You arrive on the scene of a bicycle crash involving a car. Your 10-year-old male patient was riding his bike. He is unconscious and has a great deal of bleeding from an open leg wound. Your first action should be

assure an open airway and cervical spine stabilization.

You are caring for a 6-year-old female patient who has reportedly fallen off her bed. During your exam you note small, round burns to the girl's back and several bruises of different colors on her legs and arms. Her parents state the child "falls a lot.

the child may have been abused

Once the placenta has been delivered, the EMT-Basic should

bring it to the hospital for examination.

While assisting a patient with childbirth, the EMT-Basic recognizes the presence of meconium staining. This finding indicates

The fetus may be having significant stress.

During which point of pregnancy does the fetus turn into position for normal childbirth?

The third trimester

After delivery, the EMT-Basic should inspect the perineum for

tearing and bleeding

A 4-year-old child has been seizing non-stop for 12 minutes. There is no sign of trauma, no fever, and no history of seizures. In order to assure adequate oxygenation, the EMT-Basic should immediately

begin positive pressure ventilations with oxygen

It may be difficult to feel a carotid pulse in infants because

their necks are short and fat

Which of the following is true about using a siren when responding to an emergency?

You assume that some motorists will hear your siren, yet ignore it.

You are transporting a 56-year-old male patient who called EMS due to his crushing substernal chest pain and difficulty breathing. Upon arrival at the ED your PCR is being prepared and a crew member asks about some of the terminology you are writing. The

R/O

A specialized lifting and moving technique that uses the large muscles of the legs is called a

power lift

You have just completed your assessment of an elderly male who was found on the ground. After conducting an interview of the bystanders, you determine there was little mechanism of injury involved here. When moving the patient, who has no suspected trauma

extremity lift

A device designed for moving patients over rough or irregular terrain is called a

Stokes stretcher

Which of the following is not considered by the courts and juries when evaluating negligence?

Whether the EMT-Basic asked permission to treat the patient.

Your unit is on the scene of a 45-year-old male patient who has been getting radiation treatments daily for terminal lung cancer that has spread throughout his body. As you assess the patient, it is evident that he is dying and may not survive the trip to

bargaining

Advanced directives include

do not resuscitate orders

If the spouse of an unconscious patient agrees to let the EMT-Basic treat her husband, in the absence of an advance directive, this would still be an example of _________ consent

implied

As an EMT-Basic, you must maintain up-to-date knowledge and skills. One way to accomplish this is by

attending continuing education conferences.

The infectious disease that kills more healthcare workers in the United States every year than any other disease is

hepatitis B

Your patient is lying flat on his back in bed. He is in the ______position

supine

Once the EMT-Basic has initialized care, leaving the patient without assuring the continuation of proper medical care is called:

abandonment

A system of internal and external reviews and audits of an EMS system to ensure a high quality of care is called

quality improvement

If you make an error while writing a prehospital care report, you should

draw a single horizontal line through the error, initial it, and write the correct information

Each individual box in the prehospital care report is called

a data element

When caring for a patient who does not speak the same language as you, remember to

use an interpreter, such as a family member or friend.

If you receive an order for a medication or a procedure from your medical direction, you should

repeat the order word for word

One of two basic ways a rescuer can gain access to a patient is _____, which requires no specialized tools

simple

Your unit has just arrived on the scene of a multiple car collision. One of the vehicles took down an electrical pole. As the EMT-Basic in charge of the scene, you should always consider a downed power line

to be electrically alive

The most common collisions in which ambulances are involved are those that occur:

at intersections

All of the following are considered a component of pre-run preparations, except

dispatch pre-arrival instructions

A commonly used emergency response communications resource service for hazardous material incidents is called

CHEMTREC

Once the EMS incident commander has established sectors at an MCI, a team leader is assigned to each sector and wears a ______ to be easily recognized.

highly visible reflective vest

An ambulance should be inspected every

shift

Amniotic Sac

The fluid-filled, baglike membrane in which the fetus develops

Apgar Score

A scoring system for assessing the status of a newborn that assigns a number value to each of fave areas of assessment

Birth Canal

The vagina and cervix

Bloody Show

A small amount of blood at the vagina that appears at the beginning of labor and may include a plug of pink-tinged mucus that is discharged when the cervix begins to dilate

Breech Presentation

A delivery in which the buttocks come out first

Crowning

The appearance of the infant's head at the vaginal opening during labor

Fetal Alcohol Syndrome

A condition of infants who are born to alcoholic mothers; characterized by physical and mental retardation and a variety of congenital abnormalities

Gestational Diabetes

Develops during pregnancy in women who did not have diabetes before

Limb Presentation

A delivery in which the presenting part is a single arm, leg or foot

Meconium

A dark green material in the amniotic fluid that can indicate disease in the newborn; it can be aspirated into the infant's lungs during deliver; the baby's first bowel movement

Multigravida

A woman who had had previous pregnancies

Multipara

A woman who had had more that one live birth

Nuchal Cord

An umbilical cord that is wrapped around the infant's neck

Pelvic Inflammatory Disease

An infection of the fallopian tubes and the surrounding tissues of the pelvis

Perineum

The area of skin between the vagina and the anus

Preeclampsia

A condition of late pregnancy that ivolves headache, visual changes, and swelling of the hands and feet; also called pregnancy induced hypertension

Presentation

The position in which the infant is born; the part of the infant that appears first

Primigravida

A woman who is experiencing her first pregnancy

Primipara

A woman who had had one live birth

Prolapse of the Umbilical Cord

A situation in which the umbilical cord comes out of the vagina before the infant

Spinal Bifida

A development defect in which a portion of the spinal cord of the meninges may protrude outside of the vertebrae and possibly even outside of the body, usually at the lower third of the spine in the lumbar area

Umbilical Cord

The conduit connecting mother to infant via the placenta; contains tho arteries and one vein

Vagina

The outermost cavity of a woman's reproductive system; the lower party of the birth canal

Adolescents

Children between 12 to 18 years of age

Child Abuse

Any improper or excessive action that injures or otherwise harms a child or infant; includes neglect and physical, sexual and emotional abuse

Dehydration

A state in which fluid losses are greater than fluid intake into the body ,leading to shock and death if untreated

Febrile Seizure

Seizure relation to a fever

Gastrostomy Tube

A feeding tube placed directly through the wall of the abdomen; used in patients who cannot ingest liquids or solids

Hypoventilation

Reduced minute volume, either from reduced rate and/or depth of breathing

Infancy

The first year of life

Meningitis

Inflammation of the meginges that cover the spinal cord and the brain

Neonatal

The first month after birth

Occiput

The back of the head

Pediatrics

A specialized medical practice devoted to the care of the young

Preschool-Age

Children between 3 to 6 years of age

Rigor Mortis

Stiffening of the body after death

School-Age

Children between 6 to 12 years of age

Shaken Baby Syndrome

Bleeding within the head and damage to the cervical spine of an infant who has been intentionally and forcibly shaken; a form of child abuse

Shunt

A tube that diverts excess cerebrospinal fluid from the brain to the abdomen

Sudden Infant Death Syndrome (SIDS)

Death of an infant or young child that remains unexplained after a complete autopsy

Toddler

The period following infancy until 3 years of age

Tracheostomy Tube

A tube inserted into the trachea in children who cannot breath on their own; passes through the neck directly into the major airways

Anemia

A deficiency of red blood cells or hemoglobin

Blanching

Turning white

Capillary refill time (CRT)

The amount of time that it takes for blood to return to the capillary bed after applying pressure to the skin or nailbed; indicates the status of end-organ perfusion; reliable in children younger than 6 yrs

Crackles

A crackling breath sound caused by the flow of air through liquid in the lungs; a sign of lower airway obstruction

Croup

Infection of the airway below the level of the vocal cords, usually caused by a virus

Epiglottitis

An infection of the soft tissue in the area above the vocal cords

Grunting

An 'uh' sound heard during exhalation; reflects the child's attempt to keep the alveoli open; a sign of increased work of breathing

Head Bobbing

The head lifts and tilts back during inspiration, then moves back during expiration; a sign of increased work of breathing

Nares

The external openings of the nostrils. Asingle nostril opening is called a naris

Pediatric Assessment Triangle (PAT)

A structured assessment tool that allows you to rapidly form a general impression of the infant or child without touching; consists of assessing appearance, work of breathing, and circulation to the skin

Pediatric Resuscitation Tape Measure

A tape used to estimate an infant or child's weight on the basis of length; appropriate drug doses and equipment sizes are listed on the tape

Retractions

Drawing in of the intercostal muscles and sternum during inspiration; a sign of increased work of breathing

Septum

The central divider in the nose

Stiffening Position

Optimum neutral head position for the uninjured child who require airway management

Tachypnea

Increased respiratory rate

Tenting

A condition in which the skin does not quickly return to its original shape after being pinched; indicates dehydration

Tidal Volume

The amount of air that is delivered to the lungs and airways in one inhalation

Transition Phase

A time period that allows the infant or child to become familiar with you and your equipment; only appropriate if the child's condition is stable

Advance Directives

Written doc. that specifies medical treatment for a competent patient should the patient become unable to make decisions; also called living wills

Aneurysm

A swelling or enlargement of a part of an artery, resulting from weakening of the arterial wall

Arteriosclorosis

A disease that is characterized by hardening, thickening and calcification of the arterial walls

Atherosclorosis

A disorder in which cholesterol and calcium build up inside the walls of blood vessels, forming plaque, which eventually leads to partial or complete blockage of blood flow and the formation of clots that can break off and embolize

Cataracts

Clouding of the lens of the eye or its surrounding transparent membranes

Collagen

A protein that is the chief component of connective tissues and bones

Compensated Shock

The early stage of shock, in which the body can still compensate for blood loss

Decompensated Shock

The late stage of shock, when blood pressure is falling

Delirium

A change in mental status marked by the inability to focus, think logically and maintain attention

Dementia

The slow onset of progressive disorientation, shortened attention span and loss of cognitive function

Elastin

A protein found in the elastic tissues such as skin and artery walls

Elder Abuse

Any action on the part of an older person's family member, caregiver or other associated person that takes advantage of the older individual's person, or emotional state; also called granny beating and parent battering

Hypoxia

A condition in which the body's cells and tissues do not have enough oxy

Kyphosis

A forward curling of the back caused by an abnormal increase in the curvature of the spine

Syncope

A fainting spell or transient loss of consciousness, often caused by an interruption of blood flow to the brain

Vasoconstriction

The narrowing of a blood vessel

Vasodilation

The widening of a blood vessel

Burst Fracture

Compression fracture of the vertebrae that typically results from a higher energy mechanism such as a motor vehicle crash or fall from substantial height

Central Cord Syndrome

A form of incomplete spinal cord injury in which some of the signals from the brain to the body are not received; results in weak or absent motor function, which is more pronounced in the upper extremities than the lower

Compression Fractures

Stable spinal cord injuries in which often only the anterior third of the vertebra is collapsed. This type of racture often results from minimal trauma

Polypharmacy

Simultaneous use of many medications

Septicemia

The disease state that results presence of microorganisms of their toxic products in the bloodstream

Stable Spinal Injury

A spinal injury that has a low risk of leading to permanent neurologic deficit or structural deformity

Unstable Spinal Injury

A spinal injury that has a high risk of permanent neurologic deficit of structural deformity

Casualty Collection Area

An area set up by physicians, nurses and other hospital staff near a major disaster scene where patients can receive further triage and medical care

Chemical Transportation Emergency Center (CHEMTREC)

An agency that assists emergency personnel in identifying and handling hazardous materials transport incidents

Command Post

The designated field command center where the incident commander and support personnel are located

Decontamination Area

The designated area in a hazardous materials incident where all patients and rescuers must be decontaminated before going to another area

Hazardous Materials

Any substances that are toxic, poisonous, radioactive, flammable or explosive and cause injury or death with exposure

Hazardous Materials Incident

An incident in which a hazardous material is no longer properly contained and isolated

Incident Commander

The individual who had overall command of the scene in the field

Incident Command System

An organizational system to help control, direct and coordinate emergency responders and resources; also known as an incident managment system (IMS)

Mass-Casualty Incident

An emergency situation involving three or more patients or that can place great demand on the equipment or personnel of the EMS system or had the potential to produce multiple casualties

Rehabilitation Area

The area that provides protection and treatment to fire fighters and other personnel working at an emergency. Here, workers are medically monitored and receive any needed care as they enter and leave scene

Sector Commander

The individual delegated to oversee and coordinate activity in an incident command sector; works under the incident commander

Toxicity Levels

Measures of the risk that a hazardous material poses to the health of an individual who comes into contact w/ it

Transportation Area

The area in a mass-casualty incident where ambulances and crews are organized to transport patients from the treatment area to receiving hospitals

Transportation Officer

The individual in charge of the transportation sector in a mass-casualty incident who assigns patients from the treatment area to awaiting ambulances in transportation area

Treatment Area

Location in a mass-casualty incident where patients are brought after being triaged and assigned a priority, where they are reassessed, treated and monitored until transport to the hospital

Treatment Officer

The individual, usually a physician who is in charge of and directs EMS personnel at the treatment area in mass-casualty incident

Triage Area

Designated are in a mass-casualty incident where the triage officer is located and patients are initially triaged before being taken to the treatment center

Triage Officer

The individual in charge of the incident command triage sector who directs the sorting of patients into triage categories in a mass-casualty incident

Chlorine (CL)

The first chemical agent ever used in warfare. It has a distinct odor of bleach and creates a green haze when released as a gas. Initially it produces upper airway irritation and a choking sensation

Communicability

Describes how easily a disease spreads from one human to another

Contact Hazard

A hazardous agent that gives off very little or no vapors; the skin is the primary route for this type of chemical to enter the body; also called a skin hazard

Contagious

A person infected w/ a disease that is highly communicable

Incubation

Describes the period of time from a person being exposed to a disease to the time when symptoms begin

Lymph Nodes

Area of the lymphatic system where infection fighting cells are housed

Lymphatic System

A passive circulatory system that transports a plasma like liquid called lymph, a thin fluid that bathes the tissues of the body

Miosis

Bilateral pinpoint constricted pupils

Mutagen

Substance that mutates, damages and changes the structures of DNA in the body's cells

Nerve Agents

A class of chemical called organophosphates; they function by blocking an essential enzyme in the nervous system, which causes the body's organs to become overstimulated and burn out

Neurotoxins

Biological agents that are the most deadly substances known to humans; they include botulinum toxin and ricin

Route of Exposure

Manner by which a toxic substance enters the body

Vapor Hazard

An agent that enters the body though the respiratory tract

Vesicants

Blister agents; the primary route of entry for vesicants is through the skin

Viruses

Germs that require a living host to multiply and survive

Volatility

Term used to describe how long a chemical agent will stay on a surface before it evaporates